You are on page 1of 65

Total Marks : 200

Online Prelims TEST - 8 (SUBJECT WISE)


( InsightsIAS Mock Test Series for UPSC Preliminary Exam 2020 ) Mark Scored : 88.67

1 Which of the following provisions need the consent of states for amendment in the constitution?
1. Representation of states in Parliament
2. Supreme Court and High courts
3. Elections to Parliament and state legislatures

Select the correct answer using the code given below


A. 1 and 3 only
B. 2 only
C. 2 and 3 only
D. 1 and 2 only

Your Answer : A
Correct Answer : D

Answer Justification :

By Special Majority of Parliament and Consent of States


69
55
06 of the polity can be
Those provisions of the Constitution which are related to the federal structure
8
amended by a special majority of the Parliament and also with the
8 96
consent of half of the state
legislatures by a simple majority. -
. c om
The following provisions can be amended in this way:
a il
1. Election of the President and its manner. gm
2@ and the states.
2. Extent of the executive power of the Union
4
3. Supreme Court and High courts.
j ay0 Hence Statement 2 is correct.
nia Schedule.
4. Distribution of legislative powers between the Union and the states.
s o
5. Any of the lists in the Seventh
6. Representation of i -states in Parliament. Hence Statement 1 is correct.
o n
y S
7. Power of Parliament to amend the Constitution and its procedure (Article 368 itself).
a
ElectionsAjto Parliament and State legislatures can be amended by a simple majority of the
two Houses of Parliament outside the scope of Article 368. Hence Statement 3 is incorrect.

2 Consider the following statements


1. Constituent power of Parliament under Article 368 does not enable it to alter the basic structure.
2. Power of the Parliament to amend the Constitution under Article 368 also includes the power to
amend Fundamental Rights.

Which of the statements given above is/are correct?


A. 1 only
B. 2 only
C. Both 1 and 2
D. Neither 1 nor 2

Your Answer : C
Correct Answer : C

www.insightsactivelearn.com 1
© Insights Active Learning | All rights reserved - 42423. You may not reproduce, distribute or exploit the contents in any form without
written permission by copyright owner. Copyright infringers may face civil and criminal liability
Total Marks : 200
Online Prelims TEST - 8 (SUBJECT WISE)
( InsightsIAS Mock Test Series for UPSC Preliminary Exam 2020 ) Mark Scored : 88.67

Answer Justification :

Both the statements are correct.

In the Shankari Prasad Case (1951), the constitutional validity of the First Amendment Act
(1951), which curtailed the right to property, was challenged.

The Supreme Court ruled that the power of the Parliament to amend the Constitution under Article
368 also includes the power to amend Fundamental Rights.

The present position is that the Parliament under Article 368 can amend any part of the
Constitution including the Fundamental Rights but without affecting the ‘basic structure’
of the Constitution.

However, the Supreme Court is yet to define or clarify as to what constitutes the ‘basic structure’ of
the Constitution.

3 Which of the following are the features of Parliamentary form of government? 56


9
1. Majority Party Rule 0 65
2. Dissolution of the Lower House 9 68
3. Political Homogeneity -8
m
i l .co
a
Select the correct answer using the code given below
m
@g
A. 1 and 2 only
B. 2 and 3 only 2
C. 1 only y 04
D. 1, 2 and 3 ia ja
s on
Your Answer : A ni
-
o
Correct AnswerS: D
a y
Aj
Answer Justification :

All the statements given above are correct.

Features of Parliamentary Government

Majority Party Rule: The political party which secures majority seats in the Lok Sabha forms the
government. The leader of that party is appointed as the Prime Minister by the President.

Dissolution of the Lower House: The lower house of the Parliament (Lok Sabha) can be dissolved
by the President on recommendation of the Prime Minister. In other words, the prime minister can
advise the President to dissolve the Lok Sabha before the expiry of its term and hold fresh elections.

Political Homogeneity: Usually members of the council of ministers belong to the same political
party, and hence they share the same political ideology. In case of coalition government, the
minister are bound by consensus.

www.insightsactivelearn.com 2
© Insights Active Learning | All rights reserved - 42423. You may not reproduce, distribute or exploit the contents in any form without
written permission by copyright owner. Copyright infringers may face civil and criminal liability
Total Marks : 200
Online Prelims TEST - 8 (SUBJECT WISE)
( InsightsIAS Mock Test Series for UPSC Preliminary Exam 2020 ) Mark Scored : 88.67

4 Which of the following is/are demerits of the Parliamentary System?


1. Unstable Government
2. Separation of Power
3. Government by Amateurs

Select the correct answer using the code given below


A. 1 and 2 only
B. 2 only
C. 1 and 3 only
D. 1, 2 and 3

Your Answer : C
Correct Answer : C

Answer Justification :

Parliamentary System is not conductive to administrative efficiency as the


6 9 ministers are not
5
5 of ministers.
experts in their fields. The Prime Minister has a limited choice in the selection
8 06
6
Against Separation of Powers In the parliamentary system, the9legislature and the executive are
- 8
together and inseparable. The cabinet acts as the leader of legislature as well as the executive.
m
Hence Statement 2 is incorrect.
i l .co
m a is not conductive to administrative efficiency
@g The Prime Minister has a limited choice in the
Government by Amateurs the parliamentary system
2
as the ministers are not experts in their fields.
selection of ministers; his choice is y 04
restricted to the members of Parliament alone and does not
j a
nia
extend to external talent.
o
i -s
onis/are the reasons for adopting Parliamentary System?
5 Which of the following
S
ay society
1. Nature of Indian
j
A
2. Preference to more responsibility
3. Need to avoid Legislative & executive conflicts

Select the correct answer using the code given below


A. 1 and 2 only
B. 2 and 3 only
C. 1 and 3 only
D. 1, 2 and 3

Your Answer : D
Correct Answer : D

Answer Justification :

All the statements given above is/are correct.

Nature of Indian Society India is one of the most heterogeneous States and most complex plural
societies in the world. Hence, the Constitution-makers adopted the parliamentary system as it offers
www.insightsactivelearn.com 3
© Insights Active Learning | All rights reserved - 42423. You may not reproduce, distribute or exploit the contents in any form without
written permission by copyright owner. Copyright infringers may face civil and criminal liability
Total Marks : 200
Online Prelims TEST - 8 (SUBJECT WISE)
( InsightsIAS Mock Test Series for UPSC Preliminary Exam 2020 ) Mark Scored : 88.67

greater scope for giving representation to various section, interests and regions in the government.
This promotes a national spirit among the people and builds a united India.

Preference to More Responsibility Dr B R Ambedkar pointed out in the Constituent Assembly


that ‘a democratic executive must satisfy two conditions: stability and responsibility. Unfortunately,
it has not been possible so far to devise a system which can ensure both in equal degree. The
American system gives more stability but less responsibility. The British system, on the other hand,
gives more responsibility but less stability. The Draft Constitution in recommending the
parliamentary system of
Executive has preferred more responsibility to more stability

Need to Avoid Legislative—Executive Conflicts The framers of the Constitution wanted to avoid
the conflicts between the legislature and the executive which are bound to occur in the presidential
system prevalent in USA.

6 Which of the following is/are unitary features of the Constitution?


1. Strong centre 69
2. States not indestructible 6 55
0
3. No equality of state representation
9 68
-8
om
Select the correct answer using the code given below
A. 1 and 2 only
il . c
B. 2 and 3 only a
C. 1 and 3 only @ gm
2
D. 1, 2 and 3 04
a j ay
Your Answer : D ni
Correct Answer : D - so
i
S on
ay
Answer Justification :
Aj
All the statements given above is/are correct.

Strong Centre

The division of powers is in favour of the Centre and highly inequitable from the federal angle.

Firstly, the Union List contains more subjects than the State List. Secondly, the more important
subjects have been included in the Union List. Thirdly, the Centre has overriding authority over the
Concurrent List. Finally, the residuary powers have also been left with the Centre, while in the US,
they are vested in the states. Thus, the Constitution has made the Centre very strong.

States Not Indestructible

Unlike in other federations, the states in India have no right to territorial integrity. The Parliament
can by unilateral action change the area, boundaries or name of any state. Moreover, it requires
only a simple majority and not a special majority. Hence, the Indian Federation is “an indestructible
Union of destructible states”. The American Federation, on the other hand, is described as “an

www.insightsactivelearn.com 4
© Insights Active Learning | All rights reserved - 42423. You may not reproduce, distribute or exploit the contents in any form without
written permission by copyright owner. Copyright infringers may face civil and criminal liability
Total Marks : 200
Online Prelims TEST - 8 (SUBJECT WISE)
( InsightsIAS Mock Test Series for UPSC Preliminary Exam 2020 ) Mark Scored : 88.67

indestructible Union of indestructible states”.

No Equality of State Representation

The states are given representation in the Rajya Sabha on the basis of population. Hence, the
membership varies from 1 to 31. In US, on the other hand, the principle of equality of
representation of states in the Upper House is fully recognized. Thus, the American Senate has 100
members, two
from each state. This principle is regarded as a safeguard for smaller states.

7 Consider the following statements


1. The Parliament can make laws for the whole or any part of the territory of India.
2. The laws made by a state legislature are not applicable outside the state in normal circumstance.
3. The Prime Minister is empowered to direct that an act of Parliament does not apply to a scheduled
area in the state or apply with specified modifications and exceptions.

Which of the statements given above is/are correct? 69


A. 1 and 2 only 6 55
0
B. 2 and 3 only
9 68
C. 1 only
-8
D. 2 only m
i l .co
Your Answer : A
m a
Correct Answer : A
2 @g
y 04
Answer Justification :
ia ja
o n
The Parliament can make
i - slaws for the whole or any part of the territory of India. The territory of
on the union territories, and any other area for the time being included in the
India includes the states,
S
y Hence Statement 1 is correct.
territory of India.
a
Aj
A state legislature can make laws for the whole or any part of the state. The laws made by a state
legislature are not applicable outside the state, except when there is a sufficient nexus between the
state and the object. Hence Statement 2 is correct.

The governor is empowered to direct that an act of Parliament does not apply to a scheduled area in
the state or apply with specified modifications and exceptions. Hence Statement 3 is incorrect.

8 Consider the following statements


1. In case of a conflict between the central law and the state law on a subject enumerated in the
Concurrent List, the State law prevails over the Central law.
2. The Parliament can make laws on any matter in the State list for implementing the international
treaties, agreements or conventions.

Which of the statements given above is/are correct?


A. 1 only
B. 2 only

www.insightsactivelearn.com 5
© Insights Active Learning | All rights reserved - 42423. You may not reproduce, distribute or exploit the contents in any form without
written permission by copyright owner. Copyright infringers may face civil and criminal liability
Total Marks : 200
Online Prelims TEST - 8 (SUBJECT WISE)
( InsightsIAS Mock Test Series for UPSC Preliminary Exam 2020 ) Mark Scored : 88.67

C. Both 1 and 2
D. Neither 1 nor 2

Your Answer : B
Correct Answer : B

Answer Justification :

In case of a conflict between the central law and the state law on a subject enumerated in the
Concurrent List, the Central law prevails over the State law. Hence Statement 1 is incorrect.

The Parliament can make laws on any matter in the state list for implementing the international
treaties, agreements or conventions.

This provision enables the Central government to fulfil its international obligations and
commitments. Hence Statement 2 is correct.

9
Some examples of laws enacted under the above provision are United Nations6(Privileges and
5
65 and legislations
Immunities) Act, 1947; Geneva Convention Act, 1960; Anti-Hijacking Act, 1982
0
relating to environment and TRIPS. 8 6
9
-8
9 Consider the following statements
. c om
ail by the state legislature for the consideration
1. The governor can reserve certain types of bills passed
of the President.
@ gm
42 money bills and other financial bills passed by the
2. The President can direct the states to reserve
0
ay
state legislature for his consideration.
n iaj
Which of the statements so above is/are correct?
given
-
A. 1 only
o ni
B. 2 only
y S
a
Aj1 and 2
C. Both
D. Neither 1 nor 2

Your Answer : C
Correct Answer : C

Answer Justification :

Both the statements are correct.

Besides the Parliament’s power to legislate directly on the state subjects under the exceptional
situations, the Constitution empowers the Centre to exercise control over the state’s legislative
matters in the following ways:
(i) The governor can reserve certain types of bills passed by the state legislature for the
consideration of the President. The president enjoys absolute veto over them.
(ii) Bills on certain matters enumerated in the State List can be introduced in the state legislature
only with the previous sanction of the president. (For example, the bills imposing restrictions on the
freedom of trade and commerce).

www.insightsactivelearn.com 6
© Insights Active Learning | All rights reserved - 42423. You may not reproduce, distribute or exploit the contents in any form without
written permission by copyright owner. Copyright infringers may face civil and criminal liability
Total Marks : 200
Online Prelims TEST - 8 (SUBJECT WISE)
( InsightsIAS Mock Test Series for UPSC Preliminary Exam 2020 ) Mark Scored : 88.67

(iii) The President can direct the states to reserve money bills and other financial bills passed by the
state legislature for his consideration during a financial emergency.

10 Which of the following circumstances or situation centre can give directions to the states
1. The construction and maintenance of means of communication.
2. The measures to be taken for the protection of the railways within the state.
3. Day to Day administration of State

Select the correct answer using the code given below


A. 1 and 2 only
B. 2 and 3 only
C. 3 only
D. 1, 2 and 3

Your Answer : A
Correct Answer : A
69
6 55
Answer Justification : 0
9 68
Centre’s Directions to the States -8
om
a i l.c
In addition to the above two cases, the Centre is empowered to give directions to the states with

gm
regard to the exercise of their executive power in the following matters:
@
2 of communication (declared to be of national or
(i) the construction and maintenance of4means
y 0
military importance) by the state;jaHence Statement 1 is correct.
ia
sonfor the protection of the railways within the state; Hence Statement
(ii) the measures to be taken
2 is correct. n i-
y So
ja of adequate facilities for instruction in the mother-tongue at the primary stage of
(iii) the provision
A
education to children belonging to linguistic minority groups in the state; and

(iv) the drawing up and execution of the specified schemes for the welfare of the Scheduled Tribes
in the state.

11 Consider the following statements


1. The Parliament can provide for the adjudication of any dispute or complaint with respect to the use,
distribution and control of waters of any inter-state river and river valley.
2. During the operation of a national emergency, the Centre becomes entitled to give executive
directions to a state on any matter.

Which of the statements given above is/are correct?


A. 1 only
B. 2 only
C. Both 1 and 2
D. Neither 1 nor 2

www.insightsactivelearn.com 7
© Insights Active Learning | All rights reserved - 42423. You may not reproduce, distribute or exploit the contents in any form without
written permission by copyright owner. Copyright infringers may face civil and criminal liability
Total Marks : 200
Online Prelims TEST - 8 (SUBJECT WISE)
( InsightsIAS Mock Test Series for UPSC Preliminary Exam 2020 ) Mark Scored : 88.67

Your Answer : B
Correct Answer : C

Answer Justification :

Both the statements are correct.

The Constitution contains the following provisions to secure cooperation and co-ordination between
the Centre and the states:

1. The Parliament can provide for the adjudication of any dispute or complaint with
respect to the use, distribution and control of waters of any inter-state river and river
valley.

2. The President can establish (under Article 263) an Inter-State Council to investigate and

69
discuss subject of common interest between the Centre and the states. Such a council was set
up in 1990. 5 5
0 6
9 68
During the operation of a national emergency (under Article- 8 352), the Centre becomes
. c om matter. Thus, the state governments
entitled to give executive directions to a state on ‘any’
il though they are not suspended.
are brought under the complete control of the Centre,
a
m
@g
12 Consider the following statements
0 42
j
1. The state legislature has exclusive
y
apower to levy taxes on subjects enumerated in the State List.
ia
n is vested in the State Legislature.
2. The residuary power of taxation
- so
i
on given above is/are correct?
Which of the statements
S
A. 1 onlyay
Aj
B. 2 only
C. Both 1 and 2
D. Neither 1 nor 2

Your Answer : A
Correct Answer : A

Answer Justification :

The state legislature has exclusive power to levy taxes on subjects enumerated in the State
List. Both the Parliament and the state legislature can levy taxes on subjects enumerated in the
Concurrent List. Hence Statement 1 is correct.

The residuary power of taxation (that is, the power to impose taxes not enumerated in any
of the three lists) is vested in the Parliament. Under this provision, the Parliament has imposed
gift tax, wealth tax and expenditure tax. Hence Statement 2 is incorrect.

www.insightsactivelearn.com 8
© Insights Active Learning | All rights reserved - 42423. You may not reproduce, distribute or exploit the contents in any form without
written permission by copyright owner. Copyright infringers may face civil and criminal liability
Total Marks : 200
Online Prelims TEST - 8 (SUBJECT WISE)
( InsightsIAS Mock Test Series for UPSC Preliminary Exam 2020 ) Mark Scored : 88.67

13 Consider the following statements


1. A state cannot raise any loan without the consent of the Centre, if there is still out-standing any part
of a loan made to the state by the Centre.
2. The Central government cannot make loans to any state or give guarantees in respect of loans
raised by any state.

Which of the statements given above is/are correct?


A. 1 only
B. 2 only
C. Both 1 and 2
D. Neither 1 nor 2

Your Answer : A
Correct Answer : A

Answer Justification :
69
A state cannot raise any loan without the consent of the Centre, if there
6 55 is still out-standing
80 a guarantee has been given
any part of a loan made to the state by the Centre or in respect of which
6
by the Centre. Hence Statement 1 is correct. 9
-8
The Central government can make loans to any state oro m guarantees in respect of loans raised
l . c
imaking
give
a
by any state. Any sums required for the purpose of such loans are to be charged on the
Consolidated Fund of India. Hence Statement gm2 is incorrect.
@
2
y 04
14 Which of the following is/are the a
i ja
recommendations of Sarkaria Commission?
n
1. Article 356 should be usedovery sparingly, in extreme cases as a last resort when all the available
alternatives fail. i -s
S
2. When the president onwithholds his assent to the state bills, the reasons should be communicated to
jay
the state government.
A
3. The Centre should consult the states before making a law on a subject of the Concurrent List.

Which of the statements given above is/are correct?


A. 1 only
B. 2 and 3 only
C. 1 and 3 only
D. 1, 2 and 3

Your Answer : D
Correct Answer : D

Answer Justification :

All the statements given above are correct.

Sarkaria Commission

In 1983, the Central government appointed a three-member Commission on Centre–state relations


www.insightsactivelearn.com 9
© Insights Active Learning | All rights reserved - 42423. You may not reproduce, distribute or exploit the contents in any form without
written permission by copyright owner. Copyright infringers may face civil and criminal liability
Total Marks : 200
Online Prelims TEST - 8 (SUBJECT WISE)
( InsightsIAS Mock Test Series for UPSC Preliminary Exam 2020 ) Mark Scored : 88.67

under the chairmanship of R S Sarkaria, a retired judge of the Supreme Court.

The Commission made 247 recommendations to improve Centre–state relations. The important
recommendations are mentioned below:

1. A permanent Inter-State Council called the Inter-Governmental Council should be set up under
Article 263.
2. Article 356 (President’s Rule) should be used very sparingly, in extreme cases as a last
resort when all the available alternatives fail.

3. When the president withholds his assent to the state bills, the reasons should be
communicated to the state government.

4. The Centre should have powers to deploy its armed forces, even without the consent of
states.

69
15 Consider the following statements Zonal Council
1. Zonal Councils are constitutional bodies. 5 5
2. Prime Minister is the common chairman of the five zonal councils. 806
96union territories and the Centre.
3. It aims at promoting cooperation and coordination between states,
-8
Which of the statements given above is/are correct? co
m
A. 1 and 2 only a il.
B. 3 only
@ gm
C. 2 and 3 only
0 42
D. 2 only ay j
o nia
Your Answer : B
-s
Correct Answer : Bni
S o
j a y
Answer A
Justification :

The Zonal Councils are the statutory (and not the constitutional) bodies. They are
established by an Act of the Parliament, that is, States Re-organisation Act of 1956. The act divided
the country into five zones (Northern, Central, Eastern, Western and Southern) and provided a
zonal council for each zone. Hence Statement 1 is incorrect.

The home minister of Central government is the common chairman of the five zonal
councils. Each chief minister acts as a vice-chairman of the council by rotation, holding office for a
period of one year at a time. Hence Statement 2 is incorrect.

The zonal councils aim at promoting cooperation and coordination between states, union territories
and the Centre. They discuss and make recommendations regarding matters like economic and
social planning, linguistic minorities, border disputes, inter-state transport, and so on. They are only
deliberative and advisory bodies. Hence Statement 3 is correct.

16 Consider the following statements

www.insightsactivelearn.com 10
© Insights Active Learning | All rights reserved - 42423. You may not reproduce, distribute or exploit the contents in any form without
written permission by copyright owner. Copyright infringers may face civil and criminal liability
Total Marks : 200
Online Prelims TEST - 8 (SUBJECT WISE)
( InsightsIAS Mock Test Series for UPSC Preliminary Exam 2020 ) Mark Scored : 88.67

1. During an Emergency, the Central government becomes all powerful and the states go into the total
control of the Centre.
2. The word National Emergency is mentioned in constitution

Which of the statements given above is/are correct?


A. 1 only
B. 2 only
C. Both 1 and 2
D. Neither 1 nor 2

Your Answer : A
Correct Answer : A

Answer Justification :

Both the statements are correct.

During an Emergency, the Central government becomes all powerful5and


9
6 the states go into
5
the total control of the Centre. It converts the federal structure into a6unitary one without a
0
formal amendment of the Constitution. 68 9
-8
An emergency due to war, external aggression or armed
o m rebellion (Article 352). This is
c
il. the Constitution employs the expression
popularly known as ‘National Emergency’. However,
a
m of this type. The word National Emergency is
‘proclamation of emergency’ to denote an emergency
not mentioned in constitution @ g
0 42
a j ay
ni
17 Consider the following statements
o
- s
1. The proclamation of Emergency must be approved by both the Houses of Parliament within six
i
on of its issue.
months from the date
S
j a y
2. Every resolution approving the proclamation of emergency or its continuance must be passed by
A of Parliament by a special majority.
either House

Which of the statements given above is/are correct?


A. 1 only
B. 2 only
C. Both 1 and 2
D. Neither 1 nor 2

Your Answer : B
Correct Answer : C

Answer Justification :

Both the statements are correct.

The proclamation of Emergency must be approved by both the Houses of Parliament


within one month from the date of its issue. Originally, the period allowed for approval by the
Parliament was two months, but was reduced by the 44th Amendment Act of 1978.

www.insightsactivelearn.com 11
© Insights Active Learning | All rights reserved - 42423. You may not reproduce, distribute or exploit the contents in any form without
written permission by copyright owner. Copyright infringers may face civil and criminal liability
Total Marks : 200
Online Prelims TEST - 8 (SUBJECT WISE)
( InsightsIAS Mock Test Series for UPSC Preliminary Exam 2020 ) Mark Scored : 88.67

Every resolution approving the proclamation of emergency or its continuance must be


passed by either House of Parliament by a special majority, which is, (a) a majority of the total
membership of that house, and (b) a majority of not less than two-thirds of the members of that
house present and voting.

18 Consider the following statements


1. During a national emergency, the executive power of the Centre extends to directing any state
regarding the manner in which its executive power is to be exercised.
2. The laws made by Parliament on the state subjects during a National Emergency would exist until
repealed by State Legislature or Parliament.
3. When a proclamation of national emergency is made, the six Fundamental Rights under Article 19
are automatically suspended.

Which of the statements given above is/are correct?


A. 3 only
B. 1 and 3 only
69
55
C. 1 only
D. 1 and 2 only 0 6
9 68
Your Answer : B -8
m
.co
Correct Answer : B

ai l
Answer Justification : m
2 @g
4
During a national emergency, the0executive power of the Centre extends to directing any
state regarding the manner a a y
inj which its executive power is to be exercised. In normal times,
n idirections
o
the Centre can give executive
s to a state only on certain specified matters. Hence
i-
Statement 1 is correct.
S on
ay by Parliament on the state subjects during a National Emergency become
The laws made
Aj six months after the emergency has ceased to operate. Notably, while a proclamation of
inoperative
national emergency is in operation, the President can issue ordinances on the state subjects also, if
the Parliament is not in session. Hence Statement 2 is incorrect.

According to Article 358, when a proclamation of national emergency is made, the six Fundamental
Rights under Article 19 are automatically suspended. No separate order for their suspension is
required. Hence Statement 3 is correct.

19 Consider the following statements


1. Article 359 is confined to Fundamental Rights under Article 19 only whereas Article 358 extends to
all those Fundamental Rights whose enforcement is suspended by the Presidential Order.
2. Article 358 extends to the entire country whereas Article 359 may extend to the entire country or a
part of it

Which of the statements given above is/are correct?


A. 1 only
B. 2 only

www.insightsactivelearn.com 12
© Insights Active Learning | All rights reserved - 42423. You may not reproduce, distribute or exploit the contents in any form without
written permission by copyright owner. Copyright infringers may face civil and criminal liability
Total Marks : 200
Online Prelims TEST - 8 (SUBJECT WISE)
( InsightsIAS Mock Test Series for UPSC Preliminary Exam 2020 ) Mark Scored : 88.67

C. Both 1 and 2
D. Neither 1 nor 2

Your Answer : B
Correct Answer : B

Answer Justification :

Article 358 is confined to Fundamental Rights under Article 19 only whereas Article 359 extends to
all those Fundamental Rights whose enforcement is suspended by the Presidential Order. Hence
Statement 1 is incorrect.

Article 358 extends to the entire country whereas Article 359 may extend to the entire country or a
part of it. Article 358 suspends Article 19 completely while Article 359 does not empower the
suspension of the enforcement of Articles 20 and 21. Hence Statement 2 is correct.

20 Consider the following statements


5 69
1. Article 355 imposes a duty on the Centre to ensure that the government6of5every state is carried on
0
in accordance with the provisions of the Constitution.
9 68
2. President’s rule automatically dissolves state legislature. 8 -
Which of the statements given above is/are correct? l.c om
a i
gm
A. 1 only
B. 2 only @
C. Both 1 and 2 0 42
D. Neither 1 nor 2 jay ia
son
Your Answer : C
i -
An
Correct Answer : o
S
Answer A
jay
Justification :

Article 355 imposes a duty on the Centre to ensure that the government of every state is carried on
in accordance with the provisions of the Constitution. Hence Statement 1 is correct.

It is this duty in the performance of which the Centre takes over the government of a state under
Article 356 in case of failure of constitutional
machinery in state.

President’s automatically suspends state legislature. Hence Statement 2 is incorrect.

21 Consider the following statements


1. The presidential proclamation imposing President’s Rule is subject to not subjected to judicial
review.
2. A state government pursuing anti-secular politics is liable to action under Article 356.

Which of the statements given above is/are correct?

www.insightsactivelearn.com 13
© Insights Active Learning | All rights reserved - 42423. You may not reproduce, distribute or exploit the contents in any form without
written permission by copyright owner. Copyright infringers may face civil and criminal liability
Total Marks : 200
Online Prelims TEST - 8 (SUBJECT WISE)
( InsightsIAS Mock Test Series for UPSC Preliminary Exam 2020 ) Mark Scored : 88.67

A. 1 only
B. 2 only
C. Both 1 and 2
D. Neither 1 nor 2

Your Answer : B
Correct Answer : B

Answer Justification :

The presidential proclamation imposing President’s Rule is subject to judicial review. The
satisfaction of the President must be based on relevant material. The action of the president can be
struck down by the court if it is based on irrelevant or extraneous grounds or if it was found to be
malafide or perverse. Hence Statement 1 is incorrect.

Secularism is one of the ‘basic features’ of the Constitution. Hence, a state government pursuing
anti-secular politics is liable to action under Article 356. Hence Statement 2 is correct.
69
6 55
22 Consider the following statements
6 80
1. President’s rule was never imposed on Telangana.
- 89
m for the reduction of salaries and
2. During Financial Emergency, President may issue directions
o
c
l. court.
allowances of Judges of the Supreme Court and the high
i
a
Which of the statements given above is/are@ gm
correct?
A. 1 only 0 42
B. 2 only
a j ay
i
C. Both 1 and 2
son
D. Neither 1 nor 2i -
S on
Your Answer
j ay: B
A
Correct Answer : C

Answer Justification :

Both the statements given above are correct.

Telangana is a new state and President’s rule was never imposed on it.

Effects of Financial Emergency

The consequences of the proclamation of a Financial Emergency are as follows:

1. The executive authority of the Centre extends (a) to directing any state to observe such canons of
financial propriety as are specified by it; and (b) to directions as the President may deem necessary
and adequate for the purpose.

www.insightsactivelearn.com 14
© Insights Active Learning | All rights reserved - 42423. You may not reproduce, distribute or exploit the contents in any form without
written permission by copyright owner. Copyright infringers may face civil and criminal liability
Total Marks : 200
Online Prelims TEST - 8 (SUBJECT WISE)
( InsightsIAS Mock Test Series for UPSC Preliminary Exam 2020 ) Mark Scored : 88.67

2. Any such direction may include a provision requiring

(a) the reduction of salaries and allowances of all or any class of persons serving in the state; and

(b) the reservation of all money bills or other financial bills for the consideration of the President
after they are passed by the legislature of the state.

3. The President may issue directions for the reduction of salaries and allowances of (a) all
or any class of persons serving the Union; and (b) the judges of the Supreme Court and the
high court.

23 Which of the following qualifications a person must possess to qualify for election as President?
1. He should have completed 35 years of age.
2. He should be qualified for election as a member of the Rajya Sabha
3. He should not hold any office of profit.

Which of the statements given above is/are correct?


69
A. 1 and 2 only
6 55
0
68
B. 2 and 3 only
9
-8
C. 1, 2 and 3
D. 1 and 3 only
m
i l .co
Your Answer : D
m a
Correct Answer : D
2 @g
y 04
Answer Justification :
ia ja
n
soelection as President should fulfil the following qualifications:
A person to be eligible for
-
ni of India.
1. He should be a citizen
o
S completed 35 years of age.
2. He should have
y
a
Aj be qualified for election as a member of the Lok Sabha. Hence Statement 2 is
3. He should
incorrect.
4. He should not hold any office of profit under the Union government or any state government or
any local authority or any other public authority.

A sitting President or Vice-President of the Union, the Governor of any state and a minister of the
Union or any state is not deemed to hold any office of profit and hence qualified as a presidential
candidate.

24 Which of the following powers does the President possess?


1. Executive powers
2. Judicial powers
3. Military Powers
4. Financial Powers

Select the correct answer using the code given below


A. 2 and 4 only

www.insightsactivelearn.com 15
© Insights Active Learning | All rights reserved - 42423. You may not reproduce, distribute or exploit the contents in any form without
written permission by copyright owner. Copyright infringers may face civil and criminal liability
Total Marks : 200
Online Prelims TEST - 8 (SUBJECT WISE)
( InsightsIAS Mock Test Series for UPSC Preliminary Exam 2020 ) Mark Scored : 88.67

B. 1 and 3 only
C. 1, 3 and 4 only
D. 1, 2, 3 and 4

Your Answer : D
Correct Answer : D

Answer Justification :

All the above powers are enjoyed by President.

The powers enjoyed and the functions performed by the President are:

1. Executive powers
2. Legislative powers
3. Financial powers
4. Judicial powers
5. Diplomatic powers 69
6. Military powers 6 55
0
7. Emergency powers
9 68
-8
m
25 Consider the following statements
i l .co
1. President can appoint any member of the Lok Sabha
m a to preside over its proceedings when the
2 @g
offices of both the Speaker and the Deputy Speaker fall vacant.
2. President appoints the prime minister 4and the other ministers.
3. Prime Minister hold office duringjay 0
president’s pleasure.

o nia
- sgiven above is/are correct?
Which of the statements
i
A. 2 and 3 onlyon
B. 1, 2 andy 3S
a
Aj 2 only
C. 1 and
D. None

Your Answer : C
Correct Answer : B

Answer Justification :

All the statements given above are correct.

President can appoint any member of the Lok Sabha to preside over its proceedings when
the offices of both the Speaker and the Deputy Speaker fall vacant. Similarly, he can also
appoint any member of the Rajya Sabha to preside over its proceedings when the offices of both the
Chairman and the Deputy Chairman fall vacant.

He appoints the prime minister and the other ministers. They hold office during his
pleasure.

www.insightsactivelearn.com 16
© Insights Active Learning | All rights reserved - 42423. You may not reproduce, distribute or exploit the contents in any form without
written permission by copyright owner. Copyright infringers may face civil and criminal liability
Total Marks : 200
Online Prelims TEST - 8 (SUBJECT WISE)
( InsightsIAS Mock Test Series for UPSC Preliminary Exam 2020 ) Mark Scored : 88.67

26 Consider the following statements


1. President can promulgate an ordinance only when both the Houses of Parliament are not in session
or when either of the two Houses of Parliament is not in session.
2. An ordinance like any other legislation, can be retrospective, that is, it may come into force from a
back date.

Which of the statements given above is/are correct?


A. 1 only
B. 2 only
C. Both 1 and 2
D. Neither 1 nor 2

Your Answer : A
Correct Answer : C

Answer Justification :

He can promulgate an ordinance only when both the Houses of Parliament5are 5 not in session or69
6
80 can also be issued when
when either of the two Houses of Parliament is not in session. An ordinance
6
only one House is in session because a law can be passed by both9the Houses and not by one House
alone. -8
. c om
a il
An ordinance like any other legislation, can be retrospective, that is, it may come into force from a
back date. It may modify or repeal any act ofg m
Parliament or another ordinance. It can alter or amend
a tax law also.
4 2@
j ay0
o nia
27 Consider the following statements
i - s specific procedure for the selection and appointment of the Prime
1. The Constitution provides
Minister. S on
jay have to exercise his individual judgment in the selection and appointment of the
2. President may
A
Prime Minister, when Prime Minister in office dies suddenly.

Which of the statements given above is/are correct?


A. 1 only
B. 2 only
C. Both 1 and 2
D. Neither 1 nor 2

Your Answer : B
Correct Answer : B

Answer Justification :

The Constitution does not contain any specific procedure for the selection and
appointment of the Prime Minister. Article 75 says only that the Prime Minister shall be
appointed by the president. Hence Statement 1 is incorrect.

www.insightsactivelearn.com 17
© Insights Active Learning | All rights reserved - 42423. You may not reproduce, distribute or exploit the contents in any form without
written permission by copyright owner. Copyright infringers may face civil and criminal liability
Total Marks : 200
Online Prelims TEST - 8 (SUBJECT WISE)
( InsightsIAS Mock Test Series for UPSC Preliminary Exam 2020 ) Mark Scored : 88.67

There is also one more situation when the president may have to exercise his individual
judgement in the selection and appointment of the Prime Minister, that is, when the Prime
Minister in office dies suddenly and there is no obvious successor. However, this does not imply that
the president is free to appoint any one as the Prime Minister. Hence Statement 2 is correct.

28 Consider the following statements regarding the Prime Minister


1. He allocates and reshuffles various portfolios among the ministers
2. He recommends persons who can be appointed as ministers by the president.
3. He can bring about the collapse of the council of ministers by resigning from office.

Which of the statements given above is/are correct?


A. 1 and 2 only
B. 2 only
C. 1 and 3 only
D. 1, 2 and 3

69
55
Your Answer : D
Correct Answer : D 0 6
9 68
Answer Justification : -8
. c om
All the statements given above are correct.
a il
@ gmas head of the Union council of ministers:
The Prime Minister enjoys the following powers
0 42
a j ay
1. He recommends persons
o niwho can be appointed as ministers by the president. The
s
- those persons as ministers who are recommended by the Prime Minister.
President can appoint only
i
on reshuffles various portfolios among the ministers.
2. He allocates and
3. He can askyaS
minister to resign or advise the President to dismiss him in case of difference of
opinion. Aj
a
4. He presides over the meeting of council of ministers and influences its decisions.
5. He guides, directs, controls, and coordinates the activities of all the ministers.
6. He can bring about the collapse of the council of ministers by resigning from office.

29 What is the difference in the electoral college of the Vice-President and the President of India?
1. Both the houses take part in the election of President, unlike the Vice-President who is elected by
the Rajya Sabha alone.
2. State legislative assemblies do not take part in the election of the Vice-President, unlike that of the
President.

Which of the statements given above is/are correct?


A. 1 only
B. 2 only
C. Both 1 and 2
D. Neither 1 nor 2

www.insightsactivelearn.com 18
© Insights Active Learning | All rights reserved - 42423. You may not reproduce, distribute or exploit the contents in any form without
written permission by copyright owner. Copyright infringers may face civil and criminal liability
Total Marks : 200
Online Prelims TEST - 8 (SUBJECT WISE)
( InsightsIAS Mock Test Series for UPSC Preliminary Exam 2020 ) Mark Scored : 88.67

Your Answer : B
Correct Answer : B

Answer Justification :

The Vice-President, like the president, is elected not directly by the people but by the method of
indirect election. He is elected by the members of an electoral college consisting of the members of
both Houses of Parliament. Hence Statement 1 is incorrect.

His Electoral College is different from the Electoral College for the election of the President in the
following two respects: It consists of both elected and nominated members of the Parliament (in the
case of president, only elected members).

It does not include the members of the state legislative assemblies (in the case of President, the
elected members of the state legislative assemblies are included). Hence Statement 2 is correct.

30 Which of the following is/are key features of Cabinet Committees in India?


5 69of Ministers.
65 which must be taken
1. They are not mentioned in the Constitution and the Rules of Business of Council
0
68
2. They cannot take final decisions on matters referred to them by the Cabinet,
by the Council of Ministers as a whole. 8 9
-
Which of the statements given above is/are correct? .c om
A. 1 only a il
B. 2 only
@ gm
C. Both 1 and 2
0 42
D. Neither 1 nor 2 jay ia
son
Your Answer : B
i -
Dn
Correct Answer : o
y S
Answer A j a
Justification :

They are extra-constitutional in emergence. In other words, they are not mentioned in the
Constitution. However, the Rules of Business provide for their establishment. Hence Statement 1
is incorrect.

They not only sort out issues and formulate proposals for the consideration of the Cabinet, but also
take decisions.

They are an organisational device to reduce the enormous workload of the Cabinet. They also
facilitate in-depth examination of policy issues and effective coordination. They can take final
decisions on matters referred to them by the Cabinet. However, the Cabinet can review their
decisions. Hence Statement 2 is incorrect.

31 Consider the following statements


1. The total number of ministers, including the Prime Minister, in the Council of Ministers shall not
exceed 25% of the total strength of the Lok Sabha.

www.insightsactivelearn.com 19
© Insights Active Learning | All rights reserved - 42423. You may not reproduce, distribute or exploit the contents in any form without
written permission by copyright owner. Copyright infringers may face civil and criminal liability
Total Marks : 200
Online Prelims TEST - 8 (SUBJECT WISE)
( InsightsIAS Mock Test Series for UPSC Preliminary Exam 2020 ) Mark Scored : 88.67

2. The council of ministers shall be collectively responsible to the Parliament.

Which of the statements given above is/are correct?


A. 1 only
B. 2 only
C. Both 1 and 2
D. Neither 1 nor 2

Your Answer : D
Correct Answer : D

Answer Justification :

Both the statements are incorrect.

The total number of ministers, including the Prime Minister, in the Council of Ministers shall not
exceed 15% of the total strength of the Lok Sabha. The provision was added by the 91st
9
Amendment Act of 2003. 56
0 65
8 council of ministers shall be
The ministers shall hold office during the pleasure of the President.6The
9
collectively responsible to the Lok Sabha. -8
. c om
a il
32 Which of these is/are the members of the Cabinet Committee on Economic Affairs (CCEA)?
m
1. Minister of Home Affairs
2 @g
2. Minister of External Affairs
0 4
3. Minister of Corporate Affairs jay
ia
s on
4. Minister of Road Transport and Highways
-
ni using the codes below.
Select the correctoanswer
A. 1, 2 and y 3Sonly
a
Aj 3 only
B. 1 and
C. 2 and 4 only
D. 1, 2, 3 and 4

Your Answer :
Correct Answer : D

Answer Justification :

Composition of the Cabinet Committee on Economic Affairs

Prime Minister, Minister of Home Affairs, Minister of External Affairs, Minister of Finance;
and Minister of Corporate Affairs.
Minister of Urban Development; Minister of Housing and Urban Poverty Alleviation; and
Minister of Information and Broadcasting.
Minister of Road Transport and Highways; Minister of Shipping.
Minister of Defence; Minister of Railways
Minister of Chemicals and Fertilizers; and Minister of Parliamentary Affairs.

www.insightsactivelearn.com 20
© Insights Active Learning | All rights reserved - 42423. You may not reproduce, distribute or exploit the contents in any form without
written permission by copyright owner. Copyright infringers may face civil and criminal liability
Total Marks : 200
Online Prelims TEST - 8 (SUBJECT WISE)
( InsightsIAS Mock Test Series for UPSC Preliminary Exam 2020 ) Mark Scored : 88.67

Minister of Law and Justice; and Minister of Electronics and Information Technology
Minister of Civil Aviation.
Minister of Food Processing Industries.
Minister of Agriculture and Farmers Welfare

33 Consider the following statements.


1. The President is an integral part of the Parliament.
2. The President is not required to attend the sessions of Parliament.

Which of the statements given above is/are correct?


A. 1 only
B. 2 only
C. Both 1 and 2
D. Neither 1 nor 2

Your Answer : C
Correct Answer : C
69
6 55
Answer Justification : 0
9 68
Both the statements given above are correct. -8
om
l.c and Rajya Sabha.
The Parliament consists of the President, Lok iSabha
a
m
g the Parliament, since no bill can become an act
So, the President forms an integral part
2 @of
without her consent.
y 04
ia ja
o n
But, the President is not required to attend the sessions of Parliament at all. He only
presents welcome address s
- which is passed as motion of thanks every time a new house is
i
constituted.
S on
A jay
34 Consider the following statements
1. Council of ministers is a wider body whereas cabinet is a smaller body.
2. Cabinet implements the decisions taken by the Council of Ministers.

Which of the statements given above is/are correct?


A. 1 only
B. 2 only
C. Both 1 and 2
D. Neither 1 nor 2

Your Answer : C
Correct Answer : A

Answer Justification :

www.insightsactivelearn.com 21
© Insights Active Learning | All rights reserved - 42423. You may not reproduce, distribute or exploit the contents in any form without
written permission by copyright owner. Copyright infringers may face civil and criminal liability
Total Marks : 200
Online Prelims TEST - 8 (SUBJECT WISE)
( InsightsIAS Mock Test Series for UPSC Preliminary Exam 2020 ) Mark Scored : 88.67

69
6 55
0
9 68
-8
m
i l .co
m a
@g
0 42
35 Consider the following statements jay
1. The Rajya Sabha representsn iastates and union territories of the Indian Union.
the
o
s in the Rajya Sabha are elected by the members of state legislative
2. The representatives of -states
i
on
assemblies and councils.
S
jay
Which ofAthe statements given above is/are correct?
A. 1 only
B. 2 only
C. Both 1 and 2
D. Neither 1 nor 2

Your Answer : A
Correct Answer : A

Answer Justification :

The Rajya Sabha is the Upper House (Second Chamber or House of Elders) and the Lok Sabha is
the Lower House (First Chamber or Popular House). The former represents the states and
union territories of the Indian Union, while the latter represents the people of India as a whole.
Hence Statement 1 is correct.

The representatives of states in the Rajya Sabha are elected by the


elected members of state legislative assemblies. The election is held in accordance with the

www.insightsactivelearn.com 22
© Insights Active Learning | All rights reserved - 42423. You may not reproduce, distribute or exploit the contents in any form without
written permission by copyright owner. Copyright infringers may face civil and criminal liability
Total Marks : 200
Online Prelims TEST - 8 (SUBJECT WISE)
( InsightsIAS Mock Test Series for UPSC Preliminary Exam 2020 ) Mark Scored : 88.67

system of proportional representation by means of the single transferable vote. Hence Statement
2 is incorrect.

36 Consider the following statements


1. The Constitution has empowered the Parliament to prescribe the manner of choosing the
representatives of the union territories in the Lok Sabha.
2. Loka Sabha automatically dissolves five years from the date of its first meeting after the general
elections.
3. The Constitution has fixed the term of office of members of the Rajya Sabha.

Which of the statements given above is/are correct?


A. 1 and 2 only
B. 2 and 3 only
C. 3 only
D. 1, 2 and 3

69
Your Answer : D
6 55
Correct Answer : A 0
9 68
Answer Justification : -8
om
The Constitution has empowered the Parliament
a il.cto prescribe the manner of choosing the
representatives of the union territories in m the Lok Sabha.
g
2@ Union Territories (Direct Election to the House of the
Accordingly, the Parliament has enacted the
4
0 of Lok Sabha from the union territories are also chosen by
People) Act, 1965, by which the members
y
a
aj 1 is correct.
direct election. Hence Statement
i
son
i - the Lok Sabha is not a continuing chamber. Its normal term is five
Unlike the Rajya Sabha,
n
o of its first meeting after the general elections, after which it
years from the date
S
ay dissolves. Hence Statement 2 is correct.
automatically
Aj
However, the President is authorised to dissolve the Lok Sabha at any time even before the
completion of five years and this cannot be challenged in a court of law.

The Constitution has not fixed the term of office of members of the Rajya Sabha and left it
to the Parliament. Accordingly, the Parliament in the Representation of the People Act (1951)
provided that the term of office of a member of the Rajya Sabha shall be six years. Hence
Statement 3 is incorrect.

37 Under the constitution, on which of the following criteria/s a person shall be disqualified for being
elected as a Member of Parliament?
1. if he is of unsound mind and stands so declared by a court
2. if he is an undischarged insolvent
3. He must not have been found guilty of certain election offences or corrupt practices in the elections

Select the correct answer using the code given below


A. 1 and 3 only

www.insightsactivelearn.com 23
© Insights Active Learning | All rights reserved - 42423. You may not reproduce, distribute or exploit the contents in any form without
written permission by copyright owner. Copyright infringers may face civil and criminal liability
Total Marks : 200
Online Prelims TEST - 8 (SUBJECT WISE)
( InsightsIAS Mock Test Series for UPSC Preliminary Exam 2020 ) Mark Scored : 88.67

B. 2 and 3 only
C. 1 and 2 only
D. 2 only

Your Answer : C
Correct Answer : C

Answer Justification :

Disqualifications:

Under the Constitution, a person shall be disqu-alified for being elected as a member of Parliament:

1. if he holds any office of profit under the Union or state government (except that of a minister or
any other office exempted by Parliament).
2. if he is of unsound mind and stands so declared by a court.
3. if he is an undischarged insolvent.
5 69
65of a foreign state or is
4. if he is not a citizen of India or has volun-tarily acquired the citizenship
0
under any acknowledgement of allegiance to a foreign state; and 68
5. if he is so disqualified under any law made by Parliament. 89
-
om
ai l.c disqualifications in the
The Parliament has laid down the following additional
Representation of People Act (1951):
@ gm
He must not have been found guilty4of 2 certain election offences or corrupt practices in the
0
elections. Hence Statement 3 jis
a ayincorrect.
i
son
-
38 Consider the following istatements
n
So of one House is also elected to the other House, his seat in the first House
1. If a sitting member
y
a
Aj
becomes vacant.
2. A House can declare the seat of a member vacant if he is absent from all its meetings for a period of
sixty days without its permission.

Which of the statements given above is/are correct?


A. 1 only
B. 2 only
C. Both 1 and 2
D. Neither 1 nor 2

Your Answer :
Correct Answer : C

Answer Justification :

Both the statements are correct.

If a sitting member of one House is also elected to the other House, his seat in the first

www.insightsactivelearn.com 24
© Insights Active Learning | All rights reserved - 42423. You may not reproduce, distribute or exploit the contents in any form without
written permission by copyright owner. Copyright infringers may face civil and criminal liability
Total Marks : 200
Online Prelims TEST - 8 (SUBJECT WISE)
( InsightsIAS Mock Test Series for UPSC Preliminary Exam 2020 ) Mark Scored : 88.67

House becomes vacant.

A House can declare the seat of a member vacant if he is absent from all its meetings for a period of
sixty days without its permission. In computing the period of sixty days, no account shall be taken of
any period during which the House is prorogued or adjourned for more than four consecutive days.

39 Consider the following about Half-an-hour discussions in Lok Sabha.


1. The speaker is not obliged to admit the motion if it focuses on changing an existing government
policy.
2. A member wishing to raise a matter need not give any notice in advance.

Which of the statements given above is/are correct?


A. 1 only
B. 2 only
C. Both 1 and 2
D. Neither 1 nor 2
69
Your Answer : C 6 55
0
Correct Answer : A
9 68
-8
Answer Justification :
. c om
a il in writing to the Secretary-General three
gm is desired to be raised, and shall shortly
A member wishing to raise a matter shall give notice
days in advance of the day on which the @ matter
0 42to raise: Provided that the notice shall be accompanied by
specify the point or points that he wishes
j ay for raising discussion on the matter in question. Hence
an explanatory note stating the reasons
a
i
Statement 2 is incorrect.
s on
-
iobliged
n
So since it no half-hour discussion motions are passed by the house and thus they
The speaker is not to admit the motion if it focuses on changing an existing
governmentypolicy,
a
Aj entirely on the request of the member. Hence Statement 1 is correct.
are admitted

40 Which of the following authorities adjourns the Lok Sabha House or suspends the meeting in absence
of a quorum?

A. President of India
B. Speaker, Lok Sabha
C. Leader of the House
D. Secretary, Lok Sabha

Your Answer : B
Correct Answer : B

Answer Justification :

The speaker has the following powers and duties:

www.insightsactivelearn.com 25
© Insights Active Learning | All rights reserved - 42423. You may not reproduce, distribute or exploit the contents in any form without
written permission by copyright owner. Copyright infringers may face civil and criminal liability
Total Marks : 200
Online Prelims TEST - 8 (SUBJECT WISE)
( InsightsIAS Mock Test Series for UPSC Preliminary Exam 2020 ) Mark Scored : 88.67

He maintains order and decorum in the House for conducting its business and
regulating its proceedings. This is his primary responsibility and he has final power
in this regard.
He is the final interpreter of the provisions of (a) the Constitution of India, (b) the Rules of
Procedure and Conduct of Business of Lok Sabha, and (c) the parliamentary precedents,
within the House.
He adjourns the House or suspends the meeting in absence of a quorum. The quorum to
constitute a meeting of the House is one-tenth of the total strength of the House.

41 Consider the following types of intellectual properties:


1. Geographical Indications
2. Trademarks
3. Patents

Which of the above comes under the jurisdiction of Intellectual Property Appellate Board (IPAB)?
A. 1 and 3 only
B. 3 only
C. 2 only 5 69
5
D. 1, 2 and 3 06 8
6
Your Answer : D - 89
Correct Answer : D om
ail.c
m
@g
Answer Justification :
2
04 (IPAB) exercises jurisdiction over trademarks, patents
The Intellectual Property AppellateyBoard
a
and geographical indications.iajHence, option (d) is correct.
son
n i-
The IPAB has its headquarters at Chennai. The Registry is situated at Chennai, where sittings are
also held.
y So
a
Aj
42 With reference to Co-operative societies, which of the following statements is/are correct?
1. The 99th Constitutional Amendment Act gave a constitutional status and protection to co-operative
societies.
2. Co-operative societies in constitution are mentioned in Directives Principles of State Policy.
3. Right to form a cooperative is a fundamental Right under the article 19 of the constitution.

Select the correct answer using the code given below:


A. 1 and 2 only
B. 2 only
C. 2 and 3 only
D. 1, 2 and 3

Your Answer :
Correct Answer : C

Answer Justification :

www.insightsactivelearn.com 26
© Insights Active Learning | All rights reserved - 42423. You may not reproduce, distribute or exploit the contents in any form without
written permission by copyright owner. Copyright infringers may face civil and criminal liability
Total Marks : 200
Online Prelims TEST - 8 (SUBJECT WISE)
( InsightsIAS Mock Test Series for UPSC Preliminary Exam 2020 ) Mark Scored : 88.67

The 97th Constitutional Amendment Act of 2011 gave a constitutional status and
protection to co-operative societies. Hence, statement 1 is incorrect. In this context, it made
the following three changes in the constitution:
1. It made the right to form co-operative societies a fundamental right
(Article 19). Hence, statement 3 is correct.
2. It included a new Directive Principle of State Policy on promotion of cooperative
societies (Article 43-B2). Hence, statement 2 is correct
3. It added a new Part IX-B in the Constitution which is entitled “The Cooperative Societies”
(Articles 243-ZH to 243-ZT).

National Judicial Appointments Commission (NJAC) was established by amending


the Constitution of India through the ninety-ninth constitution amendment with
the Constitution (Ninety-Ninth Amendment) Act, 2014 or 99th Constitutional
Amendment Act-2014 passed by the Lok Sabha on 13 August 2014 and by the Rajya Sabha on 14
August 2014.

43 With reference to the Model Police Act, consider the following statements: 69
6 55
0
68
1. The Act provided for social responsibilities of the police.
- 89
2. The Model Act sought earmarking dedicated staff for crime minvestigation.
o
a i l.c
gm
Which of the statements given above is/are correct?
A. 1 only @
B. 2 only 0 42
C. Both 1 and 2
a j ay
i
D. Neither 1 nor 2
son
Your Answer : C on
i-
y S: C
a
Correct Answer
Aj
Answer Justification :

A Committee set up by Ministry of Home Affairs in September, 2005 submitted a draft Model Police
Act on 30th October, 2006.

The Model Act emphasized the need to have a professional police ‘service’ in a democratic society,
which is efficient, effective, responsive to the needs of the people and accountable to the Rule of
Law. The Act provided for social responsibilities of the police and emphasizes that the
police would be governed by the principles of impartiality and human rights norms, with
special attention to protection of weaker sections including minorities. Hence, statement
1 is correct.

To ensure an efficient, responsive and professional police service, the Model Act sought
earmarking dedicated staff for crime investigation; and distinct cadre for Civil police vis-
à-vis Armed Police. Hence, statement 2 is correct.

www.insightsactivelearn.com 27
© Insights Active Learning | All rights reserved - 42423. You may not reproduce, distribute or exploit the contents in any form without
written permission by copyright owner. Copyright infringers may face civil and criminal liability
Total Marks : 200
Online Prelims TEST - 8 (SUBJECT WISE)
( InsightsIAS Mock Test Series for UPSC Preliminary Exam 2020 ) Mark Scored : 88.67

https://pib.gov.in/newsite/PrintRelease.aspx?relid=123494

44 Which of the following qualifications is/are necessary to be appointed as the District Judge?
1. He should have been an advocate or a pleader for ten years.
2. He should not already be in the service of the Central or the state government.
3. He should be recommended by the high court for appointment.

Select the correct answer using the codes given below


A. 1 and 3 only
B. 2 and 3 only
C. 3 only
D. 1 and 2 only

Your Answer : C
Correct Answer : B

69
55
Answer Justification :
06
The appointment, posting and promotion of district judges in
9 68
a state are made by the
governor of the state in consultation with the high court. -8
. c om
a il the following
gm
A person to be appointed as district judge should have
qualifications: @
2 of the Central or the state government. Hence,
(a) He should not already be in the 0 4service
statement 2 is correct.
a j ay
i
(b) He should have been an
son
advocate or a pleader for seven years. Hence, statement 1 is
incorrect.
n i-
S o
(c) He should be recommended by the high court for appointment. Hence, statement 3 is
correct. ay
Aj
45 Which of the following provisions are covered under the Representation of People Act, 1951?

1. Corrupt practices and electoral offences


2. Election Disputes
3. Registration of political parties

Select the correct answer using codes given below


A. 1 and 2 only
B. 2 and 3 only
C. 1 and 3 only
D. 1, 2 and 3

Your Answer : A
Correct Answer : D

www.insightsactivelearn.com 28
© Insights Active Learning | All rights reserved - 42423. You may not reproduce, distribute or exploit the contents in any form without
written permission by copyright owner. Copyright infringers may face civil and criminal liability
Total Marks : 200
Online Prelims TEST - 8 (SUBJECT WISE)
( InsightsIAS Mock Test Series for UPSC Preliminary Exam 2020 ) Mark Scored : 88.67

Answer Justification :

The Representation of the People Act, 1951 is an act of Parliament of India to provide for
the conduct of election of the Houses of Parliament and to the House or Houses of
the Legislature of each State, the qualifications and disqualifications for membership of
those Houses, the corrupt practices and other offences at or in connection with such
elections and the decision of doubts and disputes arising out of or in connection with such
elections. It was introduced in Parliament by law minister Dr. B.R. Ambedkar. The Act was
enacted by the provisional parliament under Article 327 of Indian Constitution, before the first
general election.

Hence, option (d) is correct.

46 Consider the following statements:


1. Under Central sector schemes, it is 100% funded by the Union government but implemented by the
State Government machinery.
69 by the States.
2. In Centrally Sponsored Scheme (CSS) a certain percentage of the funding is borne
5
65 core and optional.
3. The Central Sector Schemes have been restructured into - core of the core,
0
8
96
Which of the statements given above is/are correct?
-8
A. 2 only
. c om
B. 1 and 2 only
a il
C. 2 and 3 only m
D. 1, 2 and 3 @g 2
y 04
Your Answer : B
ia ja
n
so
Correct Answer : A
i -
on :
Answer Justification
S
j a y
Adevelopmental
In India’s plan exercise we have two types of schemes viz; central sector and
centrally sponsored scheme. The nomenclature is derived from the pattern of funding and the
modality for implementation.

Under Central sector schemes, it is 100% funded by the Union government and
implemented by the Central Government machinery. Central sector schemes are mainly
formulated on subjects from the Union List. In addition, the Central Ministries also implement some
schemes directly in States/UTs which are called Central Sector Schemes but resources under these
Schemes are not generally transferred to States. Hence, statement 1 is incorrect.

Under Centrally Sponsored Scheme (CSS) a certain percentage of the funding is borne by
the States in the ratio of 50:50, 70:30, 75:25 or 90:10 and the implementation is by the
State Governments. Centrally Sponsored Schemes are formulated in subjects from the State List
to encourage States to prioritise in areas that require more attention. Hence, statement 2 is
correct.

The Union Cabinet chaired by the Prime Minister Shri Narendra Modi has accepted the major
recommendations of the Sub-Group of Chief Minsters on Rationalization of Centrally Sponsored

www.insightsactivelearn.com 29
© Insights Active Learning | All rights reserved - 42423. You may not reproduce, distribute or exploit the contents in any form without
written permission by copyright owner. Copyright infringers may face civil and criminal liability
Total Marks : 200
Online Prelims TEST - 8 (SUBJECT WISE)
( InsightsIAS Mock Test Series for UPSC Preliminary Exam 2020 ) Mark Scored : 88.67

Schemes (CSSs).

Categorisation of Schemes: Existing CSSs should be divided into Core and Optional Schemes.

i. Core schemes: Focus of CSSs should be on schemes that comprise the National
Development Agenda where the Centre and States will work together in the spirit of Team India.

ii. Core of the Core Schemes: Those schemes which are for social protection and social
inclusion should form the core of core and be the first charge on available funds for the National
Development Agenda.

iii. Optional Schemes: The Schemes where States would be free to choose the ones they wish
to implement. Funds for these schemes would be allocated to States by the Ministry of Finance as a
lump sum.

Hence, statement 3 is incorrect.

9
http://www.arthapedia.in/index.php?title=Central_Sector_and_Centrally_Sponsored_Schemes
6
5
https://pib.gov.in/newsite/PrintRelease.aspx?relid=148299 0 65
8
96
-8
m the following statements:
47 With reference to Administrative reforms commissions, consider
o
c
l. government.
1. The recommendations given by ARC are binding onithe
a
m initially chaired by Shri Morarji R Desai.
2. The first Administrative Reforms Commission was
2 @g
4 correct?
Which of the statements given above0is/are
jay
A. 1 only
ia
B. 2 only
son
C. Both 1 and 2 i -
D. Neither 1S on2
nor
ay
Aj
Your Answer :
Correct Answer : B

Answer Justification :

The First Administrative Reforms Commission (ARC) as a Commission of Inquiry was set up in
January,1966, to examine the public administration of the country and make recommendation for
reform and reorganisation when necessary, involving the following aspects:

The machinery of the Government of India and its procedures or work


The machinery for planning at all levels
Centre- State relationships
Financial administration
Personnel administration
Economic administration
Administration at the State level
District administration

www.insightsactivelearn.com 30
© Insights Active Learning | All rights reserved - 42423. You may not reproduce, distribute or exploit the contents in any form without
written permission by copyright owner. Copyright infringers may face civil and criminal liability
Total Marks : 200
Online Prelims TEST - 8 (SUBJECT WISE)
( InsightsIAS Mock Test Series for UPSC Preliminary Exam 2020 ) Mark Scored : 88.67

Agricultural administration and


Problems of redress of citizens grievances.

The Administrative Reforms Commission was initially Chaired by Shri Morarji R Desai,
MP, and later on Shri K.Hunmanthaiya, M.P became its chairman when Shri Morarji R Desai,
MP, became the Deputy Prime Minister of India. Hence, statement 2 is correct.

The recommendations given by ARC are advisory in nature. Government can either accept or reject
the recommendations as they are non-binding. Hence, statement 1 is incorrect.

The Second Administrative Reforms Commission (ARC) was constituted on 31.08,2005, as a


Commission of Inquiry, under the Chairmanship of Shri Veerappa Moily for preparing a detailed
blueprint for revamping the public administrative system.

https://darpg.gov.in/about-arc
69
48 With reference to Citizen's Charter, consider the following statements: 06
55
9 68in the court of law.
1. The Citizen’s Charter is legally enforceable and, therefore, justiciable
- 8citizens but also all the
2. The term 'Citizen' in the Citizen's Charter includes not only the
stakeholders. om
a il.c
gm
Which of the statements given above is/are correct?
@
A. 1 only
0 42
B. 2 only
a j ay
C. Both 1 and 2 i
D. Neither 1 nor 2 son
n i-
Your Answer : CSo
ay : B
Aj
Correct Answer

Answer Justification :

Citizen’s Charter is a document which represents a systematic effort to focus on the commitment of
the Organisation towards its Citizens in respects of Standard of Services, Information, Choice and
Consultation, Non-discrimination and Accessibility, Grievance Redress, Courtesy and Value for
Money. This also includes expectations of the Organisation from the Citizen for fulfilling the
commitment of the Organisation.

The Citizen’s Charter is not legally enforceable and, therefore, is non-justiciable. However,
it is a tool for facilitating the delivery of services to citizens with specified standards, quality and
time frame etc. with commitments from the Organization and its clients. Hence, statement 1 is
incorrect.

The term ‘Citizen’ in the Citizen’s Charter implies the clients or customers whose interests
and values are addressed by the Citizen’s Charter and, therefore, includes not only the
citizens but also all the stakeholders, i.e., citizens, customers, clients, users, beneficiaries, other

www.insightsactivelearn.com 31
© Insights Active Learning | All rights reserved - 42423. You may not reproduce, distribute or exploit the contents in any form without
written permission by copyright owner. Copyright infringers may face civil and criminal liability
Total Marks : 200
Online Prelims TEST - 8 (SUBJECT WISE)
( InsightsIAS Mock Test Series for UPSC Preliminary Exam 2020 ) Mark Scored : 88.67

Ministries/ Departments/ Organisations, State Governments, UT Administrations etc. Hence,


statement 2 is correct.

https://goicharters.nic.in/faq.htm

49 With reference to the UMANG or Unified Mobile Application for New-age Governance App, consider
the following statements:
1. It is one of the key component of Digital India.
2. It is available on both android as well as iOS.
3. It is available only in Hindi and English language at present.

Which of the statements given above is/are correct?


A. 1 only
B. 1 and 2 only
C. 2 and 3 only
D. 1, 2 and 3
69
Your Answer : 6 55
0
Correct Answer : B
9 68
-8
Answer Justification :
. c om
a
The Unified Mobile Application for New-age Governance
il (UMANG), is a Government of
gm multi- platform
2 @
India all-in-one single unified secure multi-channel multi-lingual multi-
service freeware mobile app for accessing
0 4 over 1,200 central and state government services
in multiple Indian languages over y
ja Android, iOS, Windows. Hence, statement 2 is correct.
n ia
s oDigiLocker,
i-
Services such as AADHAR, Bharat Bill Payment
System, PAN, EPFOnservices, PMKVY services, AICTE, CBSE, tax and fee or utilities bills payments,
education, joby So tax, business, health, agriculture, travel, birth certificates, e-District,
search,
passportAandja much more.
This a key component of Digital India government initiative to make all traditional offline
government services available 24/7 online through single unified app. Hence, statement 1
is correct.

App is initially available in 13 Languages and will replace or compliment 1500 apps launched
by the government so far. Hence, statement 3 is incorrect.

50 With reference to Municipal corporation, consider the following statements:


1. The Municipal Council is headed by Municipal commissioner.
2. Mayor is responsible for the implementation of the decisions taken by the council and its standing
committees.

Which of the statements given above is/are correct?


A. 1 only
B. 2 only

www.insightsactivelearn.com 32
© Insights Active Learning | All rights reserved - 42423. You may not reproduce, distribute or exploit the contents in any form without
written permission by copyright owner. Copyright infringers may face civil and criminal liability
Total Marks : 200
Online Prelims TEST - 8 (SUBJECT WISE)
( InsightsIAS Mock Test Series for UPSC Preliminary Exam 2020 ) Mark Scored : 88.67

C. Both 1 and 2
D. Neither 1 nor 2

Your Answer : B
Correct Answer : D

Answer Justification :

Municipal corporations are created for the administration of big cities like Delhi, Mumbai, Kolkata,
Hyderabad, Bangalore and others. They are established in the states by the acts of the concerned
state legislatures, and in the union territories by the acts of the Parliament of India.

A municipal corporation has three authorities, namely, the council, the standing committees and the
commissioner.

The Council is headed by a Mayor. He is assisted by a Deputy Mayor. He is elected in a


majority of the states for a one-year renewable term. He is basically an ornamental figure and a
69 of the Council.
formal head of the corporation. His main function is to preside over the meetings
5
Hence, statement 1 is incorrect. 65
6 80
The municipal commissioner is responsible for the implementation
- 89 of the decisions taken
by the council and its standing committees. Thus, hem is the chief executive authority of the
c o
corporation. Hence, statement 2 is incorrect.
a il.
@ gm
51 With reference to the ‘Cantonment’ boards,
0 42which of the following statements is/are correct?
a j ay
1. It is established for municipal administration for civilian population in the cantonment area.
i control of the Ministry of Housing and Urban Affairs.
on
2. It works under the administrative
s
n i - using the codes given below:
Select the correct answer
A. 1 only y S
o
a
Aj
B. 2 only
C. Both 1 and 2
D. Neither 1 nor 2

Your Answer : A
Correct Answer : A

Answer Justification :

A cantonment board is established for municipal administration for civilian


population in the cantonment area. Hence, statement 1 is correct.

It is set up under the provisions of the Cantonments Act of 2006—a legislation enacted by the
Central government. It works under the administrative control of the defence ministry of the
Central government. Hence, statement 2 is incorrect.

52 Which among the following states in India has/have Inner Line Permit (ILP) system?

www.insightsactivelearn.com 33
© Insights Active Learning | All rights reserved - 42423. You may not reproduce, distribute or exploit the contents in any form without
written permission by copyright owner. Copyright infringers may face civil and criminal liability
Total Marks : 200
Online Prelims TEST - 8 (SUBJECT WISE)
( InsightsIAS Mock Test Series for UPSC Preliminary Exam 2020 ) Mark Scored : 88.67

1. Nagaland
2. Mizoram
3. Manipur
4. Arunachal Pradesh

Select the correct answer using the codes given below.


A. 1, 2 and 3 only
B. 1, 2 and 4 only
C. 1, 3 and 4 only
D. 1, 2 , 3 and 4

Your Answer : D
Correct Answer : B

Answer Justification :

69 for Indian
Inner Line Permit is an official travel document issued by the Government of India to allow inward
5
travel of an Indian citizen into a protected area for a limited period. It is obligatory
5
06protected state.
citizens from outside those states to obtain a permit for entering into the
9 68
Presently, ILP is required in Arunachal Pradesh, Mizoram - 8and Nagaland. Hence, option (b)
is correct.
. c om
ail
m in 2018, by the Government of Manipur. The
The bill to implement ILP in Manipur was introduced
g
4 2@ President's assent.
bill did not get passed since it could not get the

j ay0
niainvokes which of the jurisdiction under them?
53 For filing PIL, the Supreme Court
o
i -s
on
A. Advisory Jurisdiction
S
ay
B. Writ Jurisdiction
Aj jurisdiction
C. Appellate
D. Original Jurisdiction

Your Answer : D
Correct Answer : B

Answer Justification :

In India, the PIL is a product of the judicial activism role of the Supreme Court. It was introduced in
the early 1980s. Justice V.R. Krishna Iyer and Justice P.N. Bhagwati were the pioneers of the
concept of PIL.

A Public Interest Litigation can be filed before the Supreme Court under Article 32 of the
Constitution or before the High Court of a State under Article 226 of the Constitution under
their respective Writ Jurisdictions.

http://www.legalserviceindia.com/article/l171-Public-Interest-Litigation.html

www.insightsactivelearn.com 34
© Insights Active Learning | All rights reserved - 42423. You may not reproduce, distribute or exploit the contents in any form without
written permission by copyright owner. Copyright infringers may face civil and criminal liability
Total Marks : 200
Online Prelims TEST - 8 (SUBJECT WISE)
( InsightsIAS Mock Test Series for UPSC Preliminary Exam 2020 ) Mark Scored : 88.67

54 With reference to the Organization for the Prohibition of Chemical Weapons consider the following
statements:

1. It is the implementing body for the Chemical Weapons Convention.


2. It has 93 member states.

Which of the statements given above is/are correct?


A. 1 only
B. 2 only
C. Both 1 and 2
D. Neither 1 nor 2

Your Answer : A
Correct Answer : A

Answer Justification :
9
The Organisation for the Prohibition of Chemical Weapons (OPCW) is
5 56
the implementing
body for the Chemical Weapons Convention, which entered into0 6
6 8 force on 29 April 1997.
Hence, statement 1 is correct. 9
-8
The OPCW, with its 193 Member States, oversees the
. c om
global endeavour to permanently and
il
verifiably eliminate chemical weapons. Hence, statement
a2 is incorrect.
m
2 @g
55 Which of the following information is/are4generally
y 0 exempted from RTI Act?
1. Information which may constitute
ia jacontempt of court.
son
2. Information received in confidence from foreign government
3. Information which would- impede the process of investigation
i
n
So
ay answer using the code given below.
Select the correct
j
A
A. 1 and 2 only
B. 1 and 3 only
C. 2 and 3 only
D. 1, 2 and 3

Your Answer : C
Correct Answer : D

Answer Justification :

Right to Information is act of the Parliament of India to provide for setting out the practical regime
of the right to information for citizens and replaces the erstwhile Freedom of information Act, 2002.

In the RTI Act, 2005, Section 8(1) lists all of the exemptions:

(a) information, disclosure of which would prejudicially affect the security, sovereignty and integrity
of India;

www.insightsactivelearn.com 35
© Insights Active Learning | All rights reserved - 42423. You may not reproduce, distribute or exploit the contents in any form without
written permission by copyright owner. Copyright infringers may face civil and criminal liability
Total Marks : 200
Online Prelims TEST - 8 (SUBJECT WISE)
( InsightsIAS Mock Test Series for UPSC Preliminary Exam 2020 ) Mark Scored : 88.67

(b) information which may constitute contempt of court; Hence, statement 1 is correct.

(c) information that would cause a breach of privilege of Parliament or the State Legislature;

(d) information including commercial confidence, trade secrets or intellectual property, the
disclosure of which would harm the competitive position of a third party, unless the competent
authority is satisfied that larger public interest warrants the disclosure of such information;

(e) information available to a person in his fiduciary relationship, unless the competent authority is
satisfied that the larger public interest warrants the disclosure of such information;

(f) information received in confidence from foreign government; Hence, statement 2 is


correct.

(g) information, the disclosure of which would endanger the life or physical safety of any person;

(h) information which would impede the process of investigation or apprehension or


prosecution of offenders; Hence, statement 3 is correct. 9
5 56
(i) cabinet papers including records of deliberations, which come under
8 06the specified exemptions;
8 96of which has not relationship to
-
(j) information which relates to personal information the disclosure
o m
any public activity or interest, or which would cause unwarranted invasion of the privacy.
il . c
Even where requested information is covered by
a
m exemption, the information should still be
an
g
2@ in the specific case requires it.
disclosed to the applicant if the public interest
4
j ay0
56 In which of the following states
a Armed Forces (Special Powers) Act (AFSPA) is in force at
nithe
o
present?
i -s
S on
y
1. MizoramAja
2. Arunachal Pradesh
3. West Bengal
4. Assam

Select the correct answer using the code given below:


A. 2 only
B. 2 and 3 only
C. 2 and 4 only
D. 1 and 3 only

Your Answer :
Correct Answer : C

Answer Justification :

Armed Forces (Special Powers) Act (AFSPA), 1958 is an act of the Parliament of India that grant
special powers to the Indian Armed Forces the power to maintain public order in “disturbed areas”.

www.insightsactivelearn.com 36
© Insights Active Learning | All rights reserved - 42423. You may not reproduce, distribute or exploit the contents in any form without
written permission by copyright owner. Copyright infringers may face civil and criminal liability
Total Marks : 200
Online Prelims TEST - 8 (SUBJECT WISE)
( InsightsIAS Mock Test Series for UPSC Preliminary Exam 2020 ) Mark Scored : 88.67

Presently, AFSPA, 1958 is operational in entire States of Assam, Nagaland, Manipur (except
Imphal Municipal area), three districts namely Tirap, Changlang and Longding of
Arunachal Pradesh and the areas falling within the jurisdiction of the eight police stations in the
districts of Arunachal Pradesh, bordering the State of Assam. The notification declaring Manipur
and Assam as “Disturbed Areas’ have been issued by the State Governments.

Hence, option (c) is correct.

https://pib.gov.in/newsite/PrintRelease.aspx?relid=187330

57 With reference to e-Kranti (“Transforming e-Governance for Transforming Governance”), consider


the following statements:

1. The programme has been approved by the cabinet in 2018.


2. MCA 21 is a mission mode project under e-Kranti.

69
Which of the statements given above is/are correct?
6 55
0
68
A. 1 only
9
-8
B. 2 only
C. Both 1 and 2
m
D. Neither 1 nor 2
i l .co
m a
Your Answer : A
2 @g
Correct Answer : B
y 04
ia ja
Answer Justification : n
- so
ni need for transforming e-Governance and promote mobile Governance and
Considering the critical
o
S in the country, the approach and key components of e-Kranti have been
Good Governance
y
a
Ajby the Union Cabinet on 25.03.2015 with the vision of “Transforming e-
approved
Governance for Transforming Governance”. Hence, statement 1 is correct.

There are 44 Mission Mode Projects under e-Kranti programme. These mission mode projects are
grouped into Central, State and Integrated projects.

www.insightsactivelearn.com 37
© Insights Active Learning | All rights reserved - 42423. You may not reproduce, distribute or exploit the contents in any form without
written permission by copyright owner. Copyright infringers may face civil and criminal liability
Total Marks : 200
Online Prelims TEST - 8 (SUBJECT WISE)
( InsightsIAS Mock Test Series for UPSC Preliminary Exam 2020 ) Mark Scored : 88.67

69
6 55
0
9 68
Hence, statement 2 is correct.
-8
. c om
ail
58 Which of the following is/are part of World Bank "Worldwide Governance Indicators"?
1. Right to Information
@ gm
2. Political stability and lack of violence 42
3. Citizen Charters
j ay0
4. Regulatory quality
o nia
5. Rule of law
- s
i
on
6. Control of corruption
S
ay
Ajcorrect answer using the code given below.
Select the
A. 1, 2, 4 and 6 only
B. 2, 4, 5 and 6 only
C. 3, 5 and 6 only
D. 1, 2, 3, 4, 5 and 6

Your Answer : D
Correct Answer : B

Answer Justification :

The Worldwide Governance Indicators (WGI) project reports aggregate and individual
governance indicators for over 200 countries and territories over the period 1996–, for six
dimensions of governance:

Voice and Accountability


Political Stability and Absence of Violence
Government Effectiveness

www.insightsactivelearn.com 38
© Insights Active Learning | All rights reserved - 42423. You may not reproduce, distribute or exploit the contents in any form without
written permission by copyright owner. Copyright infringers may face civil and criminal liability
Total Marks : 200
Online Prelims TEST - 8 (SUBJECT WISE)
( InsightsIAS Mock Test Series for UPSC Preliminary Exam 2020 ) Mark Scored : 88.67

Regulatory Quality
Rule of Law
Control of Corruption

These aggregate indicators combine the views of a large number of enterprise, citizen and expert
survey respondents in industrial and developing countries.

Hence, option (b) is correct.

https://info.worldbank.org/governance/wgi/#home

59 With reference to the Strategy for New India @75, consider the following statements.
1. It aims to reduce the logistics cost to less than 10% of GDP from the current level of 14% by 2022
2. It was released by NITI Aayog.
3. It aims to achieve a position among top 5 countries in Global Innovation Index by 2022.

Which of the statements given above is/are correct?


69
A. 1 and 2 only
6 55
0
B. 2 and 3 only
9 68
-8
C. 1 and 3 only
D. 1, 2 and 3
m
i l .co
Your Answer :
m a
Correct Answer : A
2 @g
y 04
Answer Justification :
ia ja
o n
Drawing inspiration and sdirection from the Honourable Prime Minister‟s clarion call for
i -
establishing a Newn India by 2022, NITI Aayog embarked on a journey of formulating the
S o
ay statement 2 is correct.
„Strategy for New India @ 75‟. The document defines the strategy for 2022- 23 across forty-one
A j
areas. Hence,

Strategy aims to achieve a position among top 50 countries in Global Innovation Index by
2022. Hence, statement 3 is incorrect.

Strategy aims to reduce the logistics cost to less than 10% of GDP from the current level
of 14% by 2022. Hence, statement 1 is correct.

60 With reference to the office of the Governor, consider the following statements:
1. He should have completed the age of 30 years to qualify to the post of Governor.
2. He is appointed by the Prime minister.
3. When the same person is appointed as the governor of two or more states, the emoluments and
allowances payable to him are shared by the states in such proportion as determined by the
Parliament.

Which of the statements given above is/are not correct?


A. 1 and 2 only

www.insightsactivelearn.com 39
© Insights Active Learning | All rights reserved - 42423. You may not reproduce, distribute or exploit the contents in any form without
written permission by copyright owner. Copyright infringers may face civil and criminal liability
Total Marks : 200
Online Prelims TEST - 8 (SUBJECT WISE)
( InsightsIAS Mock Test Series for UPSC Preliminary Exam 2020 ) Mark Scored : 88.67

B. 3 only
C. 2 only
D. 1, 2 and 3

Your Answer : A
Correct Answer : D

Answer Justification :

Governor is appointed by the president by warrant under his hand and seal. Hence, statement
2 is incorrect.

The Constitution lays down only two qualifications for the appointment of a person as a governor.
These are:
1. He should be a citizen of India.
2. He should have completed the age of 35 years. Hence, statement 1 is incorrect.

When the same person is appointed as the governor of two or more states, 69
the emoluments and allowances payable to him are shared by the states6in5
5
0
such proportion as determined by the president. Hence, statement68 3 is incorrect.
9
-8
m
i l co
61 With reference to the Tribunals, consider the following .statements:

m a
1. Central Administrative Tribunal falls under the Ministry of Home Affairs.
@ g
2. The Chairman and members of the State Administrative Tribunals (SATs) are appointed by the
Governor. 42
j ay0
ia
n above is/are correct?
Which of the statements given
so
i-
A. 1 only
n
So2
B. 2 only
C. Both 1 yand
a
Aj 1 nor 2
D. Neither

Your Answer : B
Correct Answer : D

Answer Justification :

The Central Administrative Tribunal had been established under Article 323 - A of the
Constitution for adjudication of disputes and complaints with respect to recruitment and
conditions of service of persons appointed to public services and posts in connection with the
affairs of the Union or other authorities under the control of the Government.

It falls under the Ministry of personnel public grievances and pensions. Hence, statement 1 is
incorrect.

The Administrative Tribunals Act of 1985 empowers the Central government to establish the State
Administrative Tribunals (SATs) on specific request of the concerned state governments.

www.insightsactivelearn.com 40
© Insights Active Learning | All rights reserved - 42423. You may not reproduce, distribute or exploit the contents in any form without
written permission by copyright owner. Copyright infringers may face civil and criminal liability
Total Marks : 200
Online Prelims TEST - 8 (SUBJECT WISE)
( InsightsIAS Mock Test Series for UPSC Preliminary Exam 2020 ) Mark Scored : 88.67

The chairman and members of the SATs are appointed by the president after consultation
with the governor of the state concerned. Hence, statement 2 is incorrect.

62 With reference to Pressure groups, consider the following statements:


1. It acts as a liaison between the government and its members.
2. Hind Mazdoor Sabha, Bharatiya Mazdoor Sangh (BMS) and Labour Progressive Federation are
some of the Trade unions in India.

Which of the statements given above is/are correct?


A. 1 only
B. 2 only
C. Both 1 and 2
D. Neither 1 nor 2

Your Answer : C
Correct Answer : C
69
6 55
Answer Justification : 0
9 68
The term ‘pressure group’ originated in the USA. A pressure - 8 group is a group of people
who are organized actively for promoting and defending
o m their common interest. It is so
c
. by exerting pressure on the government.
called as it attempts to bring a change in the publicilpolicy
It acts as a liaison between the government a
mand its members. Hence, statement 1 is
correct. @ g
0 42
a ay
Hind Mazdoor Sabha, Bharatiya jMazdoor Sangh (BMS) and Labour Progressive Federation are
some of the Trade unions in n i
so India. Hence, statement 2 is correct.
n i-
So can be considered as the devices of Direct Democracy?
63 Which of the following
y
a
Aj
1. The Plebiscite
2. The Recall
3. The Initiative

Select the correct answer using the codes given below:


A. 1 and 3 only
B. 1 and 2 only
C. 2 and 3 only
D. 1, 2 and 3

Your Answer : D
Correct Answer : D

Answer Justification :

Democracy is of two types—direct and indirect. In direct democracy, the

www.insightsactivelearn.com 41
© Insights Active Learning | All rights reserved - 42423. You may not reproduce, distribute or exploit the contents in any form without
written permission by copyright owner. Copyright infringers may face civil and criminal liability
Total Marks : 200
Online Prelims TEST - 8 (SUBJECT WISE)
( InsightsIAS Mock Test Series for UPSC Preliminary Exam 2020 ) Mark Scored : 88.67

people exercise their supreme power directly as is the case in Switzerland.


There are four devices of direct democracy, namely, Referendum, Initiative,
Recall and Plebiscite.

Referendum is a procedure whereby a proposed legislation is referred to the electorate for


settlement by their direct votes.

Initiative is a method by means of which the people can propose a bill to the legislature for
enactment.

Recall is a method by means of which the voters can remove a representative or an officer before
the expiry of his term, when he fails to discharge his duties properly.

Plebiscite is a method of obtaining the opinion India’s of people on any issue of public importance. It
is generally used to solve the territorial disputes. 69 5
5
Hence, option (d) is correct. 8 06
6
- 89
64 With reference to the Joint Parliamentary Committee (JPC)
. c om
of India, consider the following
ail
statements:
1. A JPC is an ad-hoc committee of Parliament. gm
2@
2. It is set up for a specific object and duration.
4
3. The details regarding membership and y0 subjects are decided by President.
ja
n ia
o above is/are correct?
Which of the statement/ssgiven
A. 1 and 2 only i -
S
B. 2 and 3 onlyon
C. 1 and
A ja3yonly
D. 1, 2 and 3

Your Answer : A
Correct Answer : A

Answer Justification :

A Joint Parliamentary Committee (JPC) is an ad-hoc body. Hence, statement 1 is correct.

It is set up for a specific object and duration. Hence, statement 2 is correct.

Joint committees are set up by a motion passed in one house of Parliament and agreed to
by the other. The details regarding membership and subjects are also decided by
Parliament. For example, the motion to constitute a JPC on the stock market scam (2001) and
pesticide residues in soft drinks (2003) was moved by the government in the Lok Sabha. The motion
on the stock market scam constituted a JPC of 30 members of which 20 were from the Lok Sabha
and 10 were from the Rajya Sabha. The motion to constitute the JPC on pesticides included 10

www.insightsactivelearn.com 42
© Insights Active Learning | All rights reserved - 42423. You may not reproduce, distribute or exploit the contents in any form without
written permission by copyright owner. Copyright infringers may face civil and criminal liability
Total Marks : 200
Online Prelims TEST - 8 (SUBJECT WISE)
( InsightsIAS Mock Test Series for UPSC Preliminary Exam 2020 ) Mark Scored : 88.67

members from the Lok Sabha and 5 from the Rajya Sabha. Hence, statement 3 is incorrect.

https://www.prsindia.org/hi/theprsblog/jpc-vs-pac

65 65. Consider the following pairs:

Commission / Body Falls Under


1. Staff Selection Commission Ministry of Personnel
2. Central Bureau of Investigation Ministry of Home Affairs
3. Central Vigilance Commission Ministry of Home Affairs

Which of the pair/s given above are correctly matched?


A. 1 only
B. 2 and 3 only
C. 1 and 2 only
D. 1, 2 and 3

69
Your Answer : A
6 55
0
68
Correct Answer : A
9
Answer Justification : -8
m
i l .co
m a
2 @g
y 04
ia ja
n
- so
n i
o
a yS
Aj

www.insightsactivelearn.com 43
© Insights Active Learning | All rights reserved - 42423. You may not reproduce, distribute or exploit the contents in any form without
written permission by copyright owner. Copyright infringers may face civil and criminal liability
Total Marks : 200
Online Prelims TEST - 8 (SUBJECT WISE)
( InsightsIAS Mock Test Series for UPSC Preliminary Exam 2020 ) Mark Scored : 88.67

Hence, option (a) is correct.

66 The presiding officer (Nyayadhikari) for a Gram Nyayalaya is appointed by:

A. Chief Justice of concerned High Court.


B. Governor with the consultation of the concerned High Court.
C. State Government with the consultation of the related High Court.
D. District Judge.

Your Answer : D
Correct Answer : C

Answer Justification :

The Gram Nyayalayas Act, 2008 has been enacted to provide for the establishment
5 69 of the Gram
Nyayalayas at the grass roots level for the purposes of providing access 6to5justice to the citizens at
0
68are not denied to any citizen
their doorsteps and to ensure that opportunities for securing justice
9
due to social, economic or other disabilities. 8 -
om
ai l.c
The Gram Nyayalaya shall be court of Judicial Magistrate of the first class and its
presiding officer (Nyayadhikari) shall be appointed by the State Government in
m
consultation with the High Court. @g
0 42
Hence, option (c) is correct. ay
n iaj
- so
o ni
67 With reference to the composition of members of State Legislative Council, consider the following
statements:
y S
j
1. 1/6 th of theatotal members are elected by graduates of three years standing and residing within the
state. A
2. 1/4 th of total members are elected by the members of local bodies in the state like municipalities,
district boards, etc
3. 1/6 th of the total members are elected by teachers of three years standing in the state, not lower in
standard than secondary school.

Which of the statements given above is/are correct?


A. 1 and 2 only
B. 3 only
C. 1, 2 and 3
D. None

Your Answer : D
Correct Answer : D

Answer Justification :

www.insightsactivelearn.com 44
© Insights Active Learning | All rights reserved - 42423. You may not reproduce, distribute or exploit the contents in any form without
written permission by copyright owner. Copyright infringers may face civil and criminal liability
Total Marks : 200
Online Prelims TEST - 8 (SUBJECT WISE)
( InsightsIAS Mock Test Series for UPSC Preliminary Exam 2020 ) Mark Scored : 88.67

Unlike the members of the legislative assembly, the members of the legislative council are
indirectly elected. The maximum strength of the council is fixed at one-third of the total
strength of the assembly and the minimum strength is fixed at 40. Though the Constitution
has fixed the maximum and the minimum limits, the actual strength of a Council is fixed by
Parliament.

Of the total number of members of a legislative council:

1. 1/3 are elected by the members of local bodies in the state like municipalities, district
boards, etc., Hence, statement 1 is incorrect.

2. 1/12 are elected by graduates of three years standing and residing within the state. Hence,
statement 2 is incorrect

3. 1/12 are elected by teachers of three years standing in the state, not lower in standard than
secondary school. Hence, statement 3 is incorrect

5 69
4. 1/3 are elected by the members of the legislative assembly of the state from amongst persons
who are not members of the assembly, and 5 6
0
5. the remainder are nominated by the governor from amongst persons9 68
who have a special
- 8
knowledge or practical experience of literature, science, art, cooperative movement and social
m
service.
il. co
a
@ gm
2 about the powers of the Governor of a state?
68 Which of the following statements is/are correct
4
j ay0
1. He appoints the state election commissioner and determines his conditions of service and tenure of
office. a
nidisqualification
s
2. He decides on the questiono of of members of the state legislature in consultation
i -
with the Election Commission.
on finance commission after every five years to review the financial position of
3. He constitutes aSstate
jay and the municipalities.
the panchayats
A
4. He makes appointments, postings and promotions of the district judges in consultation with the
state high court.

Select the correct answer using the code given below.


A. 1, 3 and 4 only
B. 1, 2 and 4 only
C. 2, 3 and 4 only
D. 1, 2, 3 and 4

Your Answer : A
Correct Answer : D

Answer Justification :

The governor is the chief executive head of the state. But, like the president, he is a nominal
executive head (titular or constitutional head). The governor also acts as an agent of the central
government. Therefore, the office of governor has a dual role.

www.insightsactivelearn.com 45
© Insights Active Learning | All rights reserved - 42423. You may not reproduce, distribute or exploit the contents in any form without
written permission by copyright owner. Copyright infringers may face civil and criminal liability
Total Marks : 200
Online Prelims TEST - 8 (SUBJECT WISE)
( InsightsIAS Mock Test Series for UPSC Preliminary Exam 2020 ) Mark Scored : 88.67

Some of the functions/powers of the governor of a state:

He appoints the state election commissioner and determines his conditions of service and
tenure of office. However, the state election commissioner can be removed only in like manner
and on the like grounds as a judge of a high court. (Executive Power) Hence, statement 1 is
correct.

He decides on the question of disqualification of members of the state legislature in


consultation with the Election Commission. (Legislative Power). Hence, statement 2 is
correct.

He constitutes a finance commission after every five years to review the financial position of
the panchayats and the municipalities. (Financial Power). Hence, statement 3 is correct.

He makes appointments, postings and promotions of the district judges in consultation


with the state high court. (Judicial Power). Hence, statement 4 is correct.

9
5 56 statements:
69 With reference to the Advisory Jurisdiction of Supreme Court, consider the following
6
0President
6 8
1. The Supreme Court is not bound to give opinion on all matters that the may refer to.
2. The opinion given by the Supreme Court on matters referred to it9by the President is not binding on
the President. -8
m
i l .co
a
Which of the statements given above is/are correct?
m
A. 1 only
2 @g
04
B. 2 only
C. Both 1 and 2
jay
ia
on
D. Neither 1 nor 2
- s
Your Answer : C ni
y
Correct Answer S: oC
a
Aj
Answer Justification :

The Constitution (Article 143) authorises the president to seek the opinion of the Supreme Court in
the two categories of matters:

(a) On any question of law or fact of public importance which has arisen or which is likely to arise.

(b) On any dispute arising out of any pre-constitution treaty, agreement, covenant, engagement,
sanad or other similar instruments.11

In the first case, the Supreme Court may tender or may refuse to tender its opinion to the
president. Hence, statement 1 is correct.

But, in the second case, the Supreme Court ‘must’ tender its opinion to the president. In
both the cases, the opinion expressed by the Supreme Court is only advisory and not a
judicial pronouncement. Hence, statement 2 is correct

Hence, it is not binding on the president; he may follow or may not follow the opinion. However, it
www.insightsactivelearn.com 46
© Insights Active Learning | All rights reserved - 42423. You may not reproduce, distribute or exploit the contents in any form without
written permission by copyright owner. Copyright infringers may face civil and criminal liability
Total Marks : 200
Online Prelims TEST - 8 (SUBJECT WISE)
( InsightsIAS Mock Test Series for UPSC Preliminary Exam 2020 ) Mark Scored : 88.67

facilitates the government to have an authoritative legal opinion on a matter to be decided by it.

70 With reference to the Special officer for linguistic minorities, consider the following statements:

1. The procedure for removal of Special officer for linguistic minorities is not provided in the
Constitution.
2. The reports of Special officer for linguistic minorities are placed before the Parliament by the
President.

Which of the statements given above is/are correct?


A. 1 only
B. 2 only
C. Both 1 and 2
D. Neither 1 nor 2

Your Answer : C 69
Correct Answer : A 6 55
0
9 68
Answer Justification : -8
om
a i l.cspecify the qualifications, tenure,
It must be noted here that the Constitution does not
salaries and allowances, service conditionsm and procedure for removal of the Special
Officer for Linguistic Minorities. Hence, g
@ statement 1 is correct.
0 42
j ay falls under the Ministry of Minority Affairs. Hence, he
At the Central level, the Commissioner
a
submits the annual reports or
o niother reports to the President through the Union Minority Affairs
-
Minister. Hence, statements 2 is incorrect.
i
S on
71 Which of theja
y
A following employees are covered by the Central Administrative Tribunal (CAT)?
1. Members of the defence forces
2. Officers and servants of the Supreme Court
3. Secretarial staff of the Parliament

Select the correct answer using the code given below.


A. 1 and 3 only
B. 2 and 3 only
C. 1, 2 and 3 only
D. None

Your Answer :
Correct Answer : D

Answer Justification :

The Central Administrative Tribunal (CAT) was set up in 1985 with the principal bench at Delhi and
additional benches in different states. At present, it has 17 regular benches, 15 of which operate at

www.insightsactivelearn.com 47
© Insights Active Learning | All rights reserved - 42423. You may not reproduce, distribute or exploit the contents in any form without
written permission by copyright owner. Copyright infringers may face civil and criminal liability
Total Marks : 200
Online Prelims TEST - 8 (SUBJECT WISE)
( InsightsIAS Mock Test Series for UPSC Preliminary Exam 2020 ) Mark Scored : 88.67

the principal seats of high courts and the remaining two at Jaipur and Lucknow1. These benches
also hold circuit sittings at other seats of high courts.

The CAT exercises original jurisdiction in relation to recruitment and all service matters of public
servants covered by it. Its jurisdiction extends to the all-India services, the Central civil
services, civil posts under the Centre and civilian employees of defence services. However,
the members of the defence forces, officers and servants of the Supreme Court and the
secretarial staff of the Parliament are not covered by it.

Hence, option (d) is correct.

72 Under Article 213, the Governor of a State is empowered to:

A. Appointment of Chief Minister


B. Promulgate ordinances during the recess of the Legislature
C. Appoint the Advocate General of State
D. Exercise his pardoning powers 69
6 55
0
Your Answer : B
9 68
Correct Answer : B
-8
. c om
Answer Justification :
a il
Article 164: Appointment of Chief Minister and
@ gmCouncil of ministers to aid and advise the Governor
4 2
y 0grant
Article 161: Power of the Governor
a j a to pardons and others
i
son for the state
Article 165: Advocate-General
n i-
So of Governor to promulgate ordinances
Article 213: Power
y
ja
Hence, A option (b) is correct.

73 Consider the following statements:


#4296
1. The President can appoint duly qualified persons as additional judges of a high court for a
temporary period not exceeding five years.
2. The Governor can appoint a duly qualified person as an acting judge of a high court when a judge of
that high court (other than the chief justice) is unable to perform the duties of his office due to
absence or any other reason.

Which of the statements given above is/are correct?


A. 1 only
B. 2 only
C. Both 1 and 2
D. Neither 1 nor 2

www.insightsactivelearn.com 48
© Insights Active Learning | All rights reserved - 42423. You may not reproduce, distribute or exploit the contents in any form without
written permission by copyright owner. Copyright infringers may face civil and criminal liability
Total Marks : 200
Online Prelims TEST - 8 (SUBJECT WISE)
( InsightsIAS Mock Test Series for UPSC Preliminary Exam 2020 ) Mark Scored : 88.67

Your Answer : A
Correct Answer : D

Answer Justification :

The President can appoint duly qualified persons as additional judges of a high court for a
temporary period not exceeding two years when:
1. there is a temporary increase in the business of the high court; or
2. there are arrears of work in the high court. Hence, statement 1 is incorrect.

The President can also appoint a duly qualified person as an acting judge of a high court
when a judge of that high court (other than the chief justice) is:
1. unable to perform the duties of his office due to absence or any other
reason; or
2. appointed to act temporarily as chief justice of that high court.

An acting judge holds office until the permanent judge resumes his office. 69
6 55
0
Hence, statement 2 is incorrect.
9 68
-8
74 Consider the following statements:
. c om
a il
1. The question of disqualification under the Tenth Schedule is decided by the president of India.
m
g is subject to judicial review.
2. The disqualified made under Anti defection law
2 @
04
a j ay is/are correct?
Which of the statements given above
A. 1 only i
B. 2 only son
-
C. Both 1 and 2 ni
D. Neither 1S
o
ay
nor 2
Aj
Your Answer : B
Correct Answer : B

Answer Justification :

The question of disqualification under the Tenth Schedule is decided by the Chairman in
the case of Rajya Sabha and Speaker in the case of Lok Sabha. Hence, statement 1 is
incorrect.

In 1992, the Supreme Court ruled that the decision of the Chairman/Speaker in this regard is
subject to judicial review. Hence, statement 2 is correct.

75 Consider the following statements regarding Indian Star Tortoise


1. It is endemic to India and Pakistan only
2. It is critically endangered species
3. It is listed under the Appendix I of CITES

www.insightsactivelearn.com 49
© Insights Active Learning | All rights reserved - 42423. You may not reproduce, distribute or exploit the contents in any form without
written permission by copyright owner. Copyright infringers may face civil and criminal liability
Total Marks : 200
Online Prelims TEST - 8 (SUBJECT WISE)
( InsightsIAS Mock Test Series for UPSC Preliminary Exam 2020 ) Mark Scored : 88.67

Which of the statements given above is/are correct?


A. 1 only
B. 2 and 3 only
C. 3 only
D. 1 and 3 only

Your Answer :
Correct Answer : C

Answer Justification :

The Indian star tortoise (Geochelone elegans) is a threatened species of tortoise found in dry
areas and scrub forest in India, Pakistan and Sri Lanka. Hence, statement 1 is incorrect.

69
6 55
0
9 68
-8
m
i l .co
m a
2 @g
y 04
ia ja
n
- so
i
S on
ayVulnerable species in Red Data Book of IUCN. Hence, statement 2 is incorrect.
It is listed as
Aj
The Indian star tortoise was upgraded to CITES Appendix I in 2019 (threatened with
extinction) by full consensus among all member states (of CoP18 with 183 countries), giving it the
highest level of international protection from commercial trade. Hence, statement 3 is correct.

https://www.thehindu.com/sci-tech/science/star-tortoise-otters-get-higher-protection-at-cites/article
29281995.ece

https://en.wikipedia.org/wiki/Indian_star_tortoise

76 Consider the following statements:


1. Kodavas, a small martial community in Kodagu region of Karnataka
2. The community have its rituals around guns and weapons like swords

Which of the statements given above is/are correct?

www.insightsactivelearn.com 50
© Insights Active Learning | All rights reserved - 42423. You may not reproduce, distribute or exploit the contents in any form without
written permission by copyright owner. Copyright infringers may face civil and criminal liability
Total Marks : 200
Online Prelims TEST - 8 (SUBJECT WISE)
( InsightsIAS Mock Test Series for UPSC Preliminary Exam 2020 ) Mark Scored : 88.67

A. 1 only
B. 2 only
C. Both 1 and 2
D. Neither 1 nor 2

Your Answer :
Correct Answer : C

Answer Justification :

Kodavas, an ethno-linguistic tribe, consider themselves the original inhabitants of Kodagu district
and are known for their martial culture.

The special privilege to own firearms without a license was granted to the Kodavas, a small
martial community in Kodagu (Coorg) District in Karnataka, by the British in recognition of
their martial traditions. Hence, statement 1 is correct.

The community have its rituals around guns and weapons like swords.6 9
Hence, statement 2
5 5
is correct. 6 0
8
96
-8
https://www.thehindu.com/news/national/karnataka/exemption-to-kodavas-from-obtaining-gun-licen
ce-being-reviewed-mha-to-hc/article29087375.ece m
i l .co
m a
@ g
77 Gundla Brahmeswaram Sanctuary, often seen in news, is located in

0 42
A. Telangana
a j ay
B. Andhra Pradesh
o ni
C. Karnataka - s
D. Tamil Nadu ni
y So
a
Aj :
Your Answer
Correct Answer : B

Answer Justification :

Twenty-three tigers have so far been counted in the Gundla Brahmeswaram Wildlife Sanctuary,
located in Nallamala forests in Kurnool district. Of the 23, 17 are female and five are male and
one cub.

Hence, option (b) is correct.

https://www.thehindu.com/news/national/andhra-pradesh/23-tigers-spotted-in-gundla-brahmeswara
m-sanctuary/article27028912.ece

www.insightsactivelearn.com 51
© Insights Active Learning | All rights reserved - 42423. You may not reproduce, distribute or exploit the contents in any form without
written permission by copyright owner. Copyright infringers may face civil and criminal liability
Total Marks : 200
Online Prelims TEST - 8 (SUBJECT WISE)
( InsightsIAS Mock Test Series for UPSC Preliminary Exam 2020 ) Mark Scored : 88.67

78 Consider the following pairs


Place in News Region/Country
1.
East Kalimantan Malaysia
2.
Raqqa Syria
3.
Manama Morocco

Which of the pairs given above is/are correctly matched?


A. 1 and 2 only
B. 2 only
C. 1 and 3 only
D. 2 and 3 only

Your Answer : B
Correct Answer : B
69
Answer Justification : 6 55
0
68
89 matched.
East Kalimantan is located in Indonesia. Hence, pair 1 is incorrectly
-
.c om
https://www.thehindu.com/news/international/indonesia-to-move-capital-from-jakarta-to-east-kalima
ntan/article29260699.ece ai l
@ gm
42
Raqqa, also called Raqa, Rakka and ar-Raqqah,
0
is a city in Syria located on the northeast bank of
ay
the Euphrates River, about 160 kilometres
a j east of Aleppo. Hence, pair 2 is correctly matched.
ni city of Bahrain, with an approximate population of 157,000
olargest
Manama is the capital and s
people. Hence, pair n i3-is incorrectly matched.
y So
a
Aj following statements regarding Telecom Disputes Settlement & Appellate Tribunal
79 Consider the
(TDSAT).
1. TDSAT is a statutory body set up to adjudicate disputes and dispose of appeals with a view to
protect the interests of service providers and consumers of the telecom sector.
2. The decision of TDSAT is final and cannot be appealed in Supreme Court.

Which of the statements given above is/are correct?


A. 1 only
B. 2 only
C. Both 1 and 2
D. Neither 1 nor 2

Your Answer : A
Correct Answer : A

Answer Justification :

www.insightsactivelearn.com 52
© Insights Active Learning | All rights reserved - 42423. You may not reproduce, distribute or exploit the contents in any form without
written permission by copyright owner. Copyright infringers may face civil and criminal liability
Total Marks : 200
Online Prelims TEST - 8 (SUBJECT WISE)
( InsightsIAS Mock Test Series for UPSC Preliminary Exam 2020 ) Mark Scored : 88.67

The TRAI Act of 1997 was amended in the year 2000 and TDSAT was set up to adjudicate disputes
and dispose of appeals with a view to protect the interests of service providers and consumers of
the telecom sector. Hence, statement 1 is correct.

The decision of TDSAT is appealed to Supreme Court of India. Hence, statement 2 is incorrect.

The jurisdiction of TDSAT stands extended to matters that lay before the Cyber Appellate Tribunal
and also the Airport Economic Regulatory Authority Appellate Tribunal.

http://www.tdsat.gov.in/Delhi/Delhi.php

https://en.wikipedia.org/wiki/Telecom_Disputes_Settlement_and_Appellate_Tribunal

80 Arrange the following states in the ascending order in terms of tree plus forest cover as a percentage
of geographical area.
1. Haryana

69
2. Mizoram
3. Arunachal Pradesh
5 5
4. Bihar 06 8
6
A. 1-2-4-3 - 89
B. 2-1-3-4 om
C. 1-4-2-3 ail.c
m
@g
D. 1-4-3-2
2
Your Answer : C y 04
Correct Answer : C ia ja
n
- so
ni :
Answer Justification
o
y S
a
Aj

www.insightsactivelearn.com 53
© Insights Active Learning | All rights reserved - 42423. You may not reproduce, distribute or exploit the contents in any form without
written permission by copyright owner. Copyright infringers may face civil and criminal liability
Total Marks : 200
Online Prelims TEST - 8 (SUBJECT WISE)
( InsightsIAS Mock Test Series for UPSC Preliminary Exam 2020 ) Mark Scored : 88.67

69
6 55
0
9 68
-8
om
Hence, option (c) is correct.

il . c
m a
81 Convention no. 185 of the International LabourgOrganisation (ILO), often seen in the news, deals
@
42
with?
0
a j ayRadiations
i
A. Protection of Workers against
on
B. Equal treatment ofslabour
n i -Document
C. Seafarer Identity
Soabove
D. None of the
y
A ja
Your Answer :
Correct Answer : C

Answer Justification :

India has become the first country in the world to issue Biometric Seafarer Identity Document
(BSID), capturing the facial bio-metric data of seafarers.

The new card is in confirmation of the Convention No. 185 of the International Labour Organisation
on BSID.

Hence, option (c) is correct.

https://pib.gov.in/PressReleaseIframePage.aspx?PRID=1583253

82 Consider the following statements regarding Amazon forest


1. The majority of the forest is contained within Brazil, with 60% of the rain forest, followed by Peru

www.insightsactivelearn.com 54
© Insights Active Learning | All rights reserved - 42423. You may not reproduce, distribute or exploit the contents in any form without
written permission by copyright owner. Copyright infringers may face civil and criminal liability
Total Marks : 200
Online Prelims TEST - 8 (SUBJECT WISE)
( InsightsIAS Mock Test Series for UPSC Preliminary Exam 2020 ) Mark Scored : 88.67

2. Amazon rainforest spreads through more than five countries

Which of the statements given above is/are correct?


A. 1 only
B. 2 only
C. Both 1 and 2
D. Neither 1 nor 2

Your Answer : B
Correct Answer : C

Answer Justification :

The Amazon rainforest is a moist broadleaf tropical rainforest in the Amazon biome that covers
most of the Amazon basin of South America. This basin encompasses 7,000,000 km2 (2,700,000 sq
mi), of which 5,500,000 km2 (2,100,000 sq mi) are covered by the rainforest. This region includes
territory belonging to nine nations. Hence, both statement 1 and statement 2 are correct.
69
55
06
https://www.thehindu.com/opinion/editorial/earths-burning-lungs-on-amazons-rainforest-fire/article2
8
9262476.ece. 6 9
-8
https://en.wikipedia.org/wiki/Amazon_rainforest m
i l .co
m a
83 Consider the following statements
@g
42 of the Chief Information Commissioner are determined
1. Salary and terms and conditions of service
0
by Supreme Court
a j ay
ni
2. Right to Information is a fundamental right
- so
i
on given above is/are correct?
Which of the statements
A. 1 only y S
a
Aj
B. 2 only
C. Both 1 and 2
D. Neither 1 nor 2

Your Answer : B
Correct Answer : B

Answer Justification :

Salary and terms and conditions of service of the Chief Information Commissioner is now
determined by Central Government. Hence, statement 1 is incorrect.

Right to Information is a fundamental right guaranteed under Part 3, Article 19(1)(a) of the
Constitution of India. Hence, statement 2 is correct.

https://www.thehindu.com/opinion/editorial/undermining-rti/article28701717.ece

www.insightsactivelearn.com 55
© Insights Active Learning | All rights reserved - 42423. You may not reproduce, distribute or exploit the contents in any form without
written permission by copyright owner. Copyright infringers may face civil and criminal liability
Total Marks : 200
Online Prelims TEST - 8 (SUBJECT WISE)
( InsightsIAS Mock Test Series for UPSC Preliminary Exam 2020 ) Mark Scored : 88.67

84 Momentum for Change is an initiative of

A. UN Climate Change secretariat


B. UNEP
C. World Bank
D. UNICEF

Your Answer : B
Correct Answer : A

Answer Justification :

Momentum for Change is an initiative spearheaded by the UN Climate Change secretariat


to shine a light on the enormous groundswell of activities underway across the globe that are
moving the world toward a highly resilient, low-carbon future.

Momentum for Change recognizes innovative and transformative solutions that9


address both
climate change and wider economic, social and environmental challenges.556
0 6
Hence, option (a) is correct. 9 68
-8
om
i .c
85 Consider the following statements regarding Group 7 l(G7)
a
1. Canada, China, Germany are members of G7 m
@g in 1990’s
2. It is an intergovernmental organisation formed
2
3. Russia, recently joined the G7 group 04
a j ay
i
on above is/are correct?
Which of the statements given
s
A. 1 and 2 only i -
B. 2 and 3 onlyon
S
C. 2 onlyay
Aj
D. None

Your Answer : B
Correct Answer : D

Answer Justification :

The Group of Seven (G7) is an international intergovernmental economic organization consisting of


the seven largest IMF-described advanced economies in the world: Canada, France, Germany,
Italy, Japan, the United Kingdom, and the United States. Hence, statement 1 is incorrect.

The concept of a forum for the world's major industrialized countries emerged before the 1973 oil
crisis. On Sunday, 25 March 1973, the U.S. Secretary of the Treasury, George Shultz, convened an
informal gathering of finance ministers from West Germany Helmut Schmidt, France Valéry Giscard
d'Estaing, and the United Kingdom Anthony Barber before an upcoming meeting in Washington,
D.C. Hence, statement 2 is incorrect.

In March 2014 the Russian Federation was suspended by G7 members from the political forum G8
www.insightsactivelearn.com 56
© Insights Active Learning | All rights reserved - 42423. You may not reproduce, distribute or exploit the contents in any form without
written permission by copyright owner. Copyright infringers may face civil and criminal liability
Total Marks : 200
Online Prelims TEST - 8 (SUBJECT WISE)
( InsightsIAS Mock Test Series for UPSC Preliminary Exam 2020 ) Mark Scored : 88.67

following the annexation of Crimea. After the suspension, on January 2017 the Russian Federation
decided permanently to leave the G8. It was confirmed in June 2018. Hence, statement 3 is
incorrect.

https://www.thehindu.com/opinion/editorial/message-delivered/article29282081.ece

86 Mitra Crater, sometimes seen in the news, is located in

A. Mars
B. Moon
C. Venus
D. Antarctica

Your Answer : B
Correct Answer : B

Answer Justification : 69
6 55
80of the larger crater Mach,
Mitra is a lunar impact crater that is attached to the western outer rim
6
89and to the south-southeast lies
on the far side of the Moon. Just to the west of Mitra is Bredikhin,
-
Henyey. It is named after Sisir Kumar Mitra. Hence, option m (b) is correct.
i l .co
a
https://www.thehindu.com/sci-tech/science/chandrayaan-2-scans-mitra-crater/article29262668.ece
m
2 @g
87 Consider the following pairs: y 04
Geographical Feature ia ja Region
n
so Separates
th
1. 38 Parallel North and South Korea
2. Strait of Malacca i - Connects Bay of Bengal and East China Sea
S on
3. Strait of Gibraltar Connects Mediterranean sea and Red sea
ay
Aj
Which of the above pair is correctly matched?
A. 1 only
B. 1 and 2 only
C. 3 only
D. None

Your Answer : A
Correct Answer : A

Answer Justification :

Strait of Malacca connects Bay of Bengal and South China Sea and Strait of Gibraltar connects
Mediterranean and Atlantic Ocean.

Hence, option (a) is correct.

88 Global Wage Report is released by

www.insightsactivelearn.com 57
© Insights Active Learning | All rights reserved - 42423. You may not reproduce, distribute or exploit the contents in any form without
written permission by copyright owner. Copyright infringers may face civil and criminal liability
Total Marks : 200
Online Prelims TEST - 8 (SUBJECT WISE)
( InsightsIAS Mock Test Series for UPSC Preliminary Exam 2020 ) Mark Scored : 88.67

A. World Bank
B. International Monetary Fund
C. International Labour Organisation
D. United Nations Development Programme

Your Answer : C
Correct Answer : C

Answer Justification :

Global Wage Report is released by International Labour Organisation (ILO). This new ILO Global
Wage Report, 2018-19 examines the evolution of real wages around the world, giving a unique
picture of wage trends globally and by region.

Hence, option (c) is correct.

https://www.ilo.org/global/research/global-reports/global-wage-report/lang--en/index.htm
69
6 55
0
9 68
89 Open Acreage Licensing, often seen in news, is related to -8
m
i l .co
A. Hydrocarbon exploration
m a
B. Railways
2 @g
04
C. Mining of Iron ore
D. Land Acquisition
jay
n ia
o
Your Answer : A -s
Correct Answer : Ani
S o
j a y
Answer A
Justification :

What is Open Acreage Licensing Policy (OALP)?

The OALP, a critical part of the Hydrocarbon Exploration and Licensing Policy, provides uniform
licences for exploration and production of all forms of hydrocarbons, enabling contractors to
explore conventional as well as unconventional oil and gas resources.

Fields are offered under a revenue-sharing model and throw up marketing and pricing freedom for
crude oil and natural gas produced.

Hence, option (a) is correct.

https://www.insightsonindia.com/2019/08/27/open-acreage-licensing-policy-3/

90 With reference to the International Energy Forum (IEF), consider the following statements:
1. It is an inter-governmental organization
2. India is not a member country of International Energy Forum

www.insightsactivelearn.com 58
© Insights Active Learning | All rights reserved - 42423. You may not reproduce, distribute or exploit the contents in any form without
written permission by copyright owner. Copyright infringers may face civil and criminal liability
Total Marks : 200
Online Prelims TEST - 8 (SUBJECT WISE)
( InsightsIAS Mock Test Series for UPSC Preliminary Exam 2020 ) Mark Scored : 88.67

Which of the statements given above is/are correct?


A. 1 only
B. 2 only
C. Both 1 and 2
D. Neither 1 nor 2

Your Answer :
Correct Answer : A

Answer Justification :

About International Energy Forum (IEF):


1. The International Energy Forum (IEF) aims to foster greater mutual understanding and
awareness of common energy interests among its members.
2. The 72 Member Countries of the Forum are signatories to the IEF Charter, which outlines the
framework of the global energy dialogue through this inter-governmental arrangement.
3. Covering all six continents and accounting for around 90% of global supply and demand for
9
6producing
oil and gas, the IEF is unique in that it comprises not only consuming and 5
countries of the International Energy Agency and OPEC, but also0Transit 65 States and major
players outside of their memberships, including Argentina, China, 9 68
India, Mexico, Russia and
South Africa. - 8
4. The IEF is the neutral facilitator of informal, open, o m
informed and continuing global energy
i l .c
dialogue.
m a
5. The Forum's biennial Ministerial Meetings
@ g are the world's largest gathering of Energy
Ministers.
0 42are promoted by a permanent Secretariat of
jay
6.The IEF and the global energy dialogue
international staff based in theia
Diplomatic Quarter of Riyadh, Saudi Arabia.
son
i -
https://www.ief.org/about-ief/ief-overview.aspx
o n
a yS
https://en.trend.az/business/energy/3113229.html
Aj
91 Consider the following statements:
1. The period spanning between the prorogation of a House of parliament and its reassembly in a new
session is called ‘recess’.
2. When the House of parliament is adjourned without naming a day for reassembly, it is called
adjournment sine die.
3. President can prorogue the House while in session.

Which of the statements given above is/are correct?


A. 1 and 2 only
B. 2 only
C. 1 and 3 only
D. 1, 2 and 3

Your Answer : D
Correct Answer : D

www.insightsactivelearn.com 59
© Insights Active Learning | All rights reserved - 42423. You may not reproduce, distribute or exploit the contents in any form without
written permission by copyright owner. Copyright infringers may face civil and criminal liability
Total Marks : 200
Online Prelims TEST - 8 (SUBJECT WISE)
( InsightsIAS Mock Test Series for UPSC Preliminary Exam 2020 ) Mark Scored : 88.67

Answer Justification :

A ‘session’ of Parliament is the period spanning between the first sitting of a House and its
prorogation (or dissolution in the case of the Lok Sabha). During a session, the House meets
everyday to transact business. The period spanning between the prorogation of a House and
its reassembly in a new session is called ‘recess’.

Adjournment sine die means terminating a sitting of Parliament for an indefinite period. In
other words, when the House is adjourned without naming a day for reassembly, it is called
adjournment sine die. The power of adjournment as well as adjournment sine die lies with
the presiding officer of the House.

The presiding officer (Speaker or Chairman) declares the House adjourned sine die, when the
business of a session is completed. Within the next few days, the President issues a
notification for prorogation of the session. However, the President can also prorogue the
House while in session.

69
92 Consider the following statements about ‘Law Commission’:
6 55
1. It is an statutory body created by the government from time to time. 80
6 every three years.
89
2. The Commission was originally constituted in 1955 and is reconstituted
-
Select the correct answer using the codes given below .c om
A. 1 only ai l
B. 2 only @ gm
C. Both 1 and 2
0 42
D. Neither 1 nor 2 jay ia
son
Your Answer : C
i -
Bn
Correct Answer : o
y S
Answer A j a
Justification :

Law Commission of India is an executive body established by an order of the Government


of India. Its major function is to work for legal reform. Its membership primarily comprises legal
experts, who are entrusted a mandate by the Government. Hence, statement 1 is incorrect.

The Government of India reacted favourably and established the First Law Commission of
Independent India in 1955 with the then Attorney-General of India, Mr. M. C. Setalvad, as its
Chairman. Since then twenty one more Law Commissions have been appointed, each with a
three-year term and with different terms of reference. Hence, statement 2 is correct.

93 The Budapest Convention is related to which of the following matters?

A. Organic Farming
B. Ozone Depletion
C. World Trade Organization
D. Cyber Security

www.insightsactivelearn.com 60
© Insights Active Learning | All rights reserved - 42423. You may not reproduce, distribute or exploit the contents in any form without
written permission by copyright owner. Copyright infringers may face civil and criminal liability
Total Marks : 200
Online Prelims TEST - 8 (SUBJECT WISE)
( InsightsIAS Mock Test Series for UPSC Preliminary Exam 2020 ) Mark Scored : 88.67

Your Answer : D
Correct Answer : D

Answer Justification :

The Convention on Cybercrime of the Council of Europe (CETS No.185), known as the Budapest
Convention, is the only binding international instrument on this issue. It serves as a guideline for
any country developing comprehensive national legislation against Cybercrime and as a framework
for international cooperation between State Parties to this treaty.

The Budapest Convention is supplemented by a Protocol on Xenophobia and Racism committed


through computer systems.

India is not a member to the convention at present. However, recently Ministry of Home affairs
mentioned that India is reconsidering its position on becoming a member of the Budapest
Convention because of the surge in cybercrime, especially after a push for digital India. Hence,
option (d) is correct.
69
https://www.coe.int/en/web/cybercrime/the-budapest-convention
6 55
0
68
-
94 With reference to the Special Leave to appeal by the Supreme Court,
89
consider the following
m
statements:
i l . co
m a
1. The Supreme Court is authorized to grant in its discretion special leave to appeal from any

@ g
judgment in any matter passed by any court or tribunal in the country excluding military tribunal
and court martial.
0 42
2. It is a discretionary power. y ja
n ia
Which of the statements so above is/are correct?
given
-
A. 1 only
o ni
B. 2 only
y S
a
Aj1 and 2
C. Both
D. Neither 1 nor 2

Your Answer : B
Correct Answer : C

Answer Justification :

The Supreme Court is authorised to grant in its discretion special leave to appeal from any
judgement in any matter passed by any court or tribunal in the country (except military
tribunal and court martial). Hence, statement 1 is correct.

This provision contains the four aspects as under:


1. It is a discretionary power and hence, cannot be claimed as a matter of
right. Hence, statement 2 correct.
2. It can be granted in any judgement whether final or interlocutory.
3. It may be related to any matter—constitutional, civil, criminal, income tax, labour, revenue,
advocates, etc.

www.insightsactivelearn.com 61
© Insights Active Learning | All rights reserved - 42423. You may not reproduce, distribute or exploit the contents in any form without
written permission by copyright owner. Copyright infringers may face civil and criminal liability
Total Marks : 200
Online Prelims TEST - 8 (SUBJECT WISE)
( InsightsIAS Mock Test Series for UPSC Preliminary Exam 2020 ) Mark Scored : 88.67

4. It can be granted against any court or tribunal and not necessarily against a high court
(except a military court).

95 With reference to the Speaker Pro Tem, consider the following statements:
1. President of India administers oath to the Speaker Pro Tem.
2. The Speaker Pro Tem has all the powers of the Speaker.

Which of the statements given above is/are correct?


A. 1 only
B. 2 only
C. Both 1 and 2
D. Neither 1 nor 2

Your Answer : C
Correct Answer : C

69
55
Answer Justification :

As provided by the Constitution, the Speaker of the last Lok Sabha6 8


vacates
06
his office immediately
9
- 8 the President appoints a
before the first meeting of the newly-elected Lok Sabha. Therefore,
member of the Lok Sabha as the Speaker Pro Tem. Usually,
o m the senior most member is
c
il. oath to the Speaker Pro Tem. Hence,
selected for this. The President himself administers
statement 1 is correct. m a
2 @g
The Speaker Pro Tem has all the powers
y 04 of the Speaker. Hence, statement 2 is correct.
ia ja
n of the newly-elected Lok Sabha. His main duty is to administer
He presides over the first sitting
o
-
oath to the new members. s He also enables the House to elect the new Speaker. When the
new Speaker is o
i
n by the House, the office of the Speaker Pro Tem ceases to exist.
elected
S
ay
Aj
96 Which of the following Countries is not part of The Bay of Bengal Initiative for Multi-Sectoral
Technical and Economic Cooperation (BIMSTEC)?

A. Bangladesh
B. Nepal
C. Thailand
D. Cambodia

Your Answer : D
Correct Answer : D

Answer Justification :

The Bay of Bengal Initiative for Multi-Sectoral Technical and Economic Cooperation is an
international organisation of seven nations of South Asia and South East Asia, housing 1.5 billion
people and having a combined gross domestic product of $3.5 trillion.

www.insightsactivelearn.com 62
© Insights Active Learning | All rights reserved - 42423. You may not reproduce, distribute or exploit the contents in any form without
written permission by copyright owner. Copyright infringers may face civil and criminal liability
Total Marks : 200
Online Prelims TEST - 8 (SUBJECT WISE)
( InsightsIAS Mock Test Series for UPSC Preliminary Exam 2020 ) Mark Scored : 88.67

The BIMSTEC member states—Bangladesh, India, Myanmar, Sri Lanka, Thailand , Nepal and
Bhutan—are among the countries dependent on the Bay of Bengal.

Hence, option (d) is correct.

https://www.thehindu.com/news/national/union-defence-minister-on-a-three-day-visit-to-south-korea/
article29340992.ece

97 Pelindaba Treaty, sometimes seen in news, is related to?

A. World Trade Organization


B. Climate Refugees
C. Nuclear Weapon Free Zone
D. Protection of Endangered species

Your Answer : B
Correct Answer : C 69
6 55
0
Answer Justification : 68
- 89
The African Nuclear Weapon Free Zone Treaty, also known
o m as the Treaty of Pelindaba (named
i .
after South Africa's main Nuclear Research Centre, lrun c by The South African Nuclear Energy
a
Corporation and was the location where South Africa's atomic bombs of the 1970s were developed,
@ gm a Nuclear-Weapon-Free Zone in Africa. The
constructed and subsequently stored), establishes
treaty was signed in 1996 and came into
0 42effect with the 28th ratification on 15 July 2009.
a j ay
i
on
Hence, option (c) is correct.
s
n i-
o
98 Which among theSfollowing countries established the Eastern Economic Forum (EEF)?
a y
A.
Aj India
B. China
C. Russia
D. Japan

Your Answer : C
Correct Answer : C

Answer Justification :

The Eastern Economic Forum was established by decree of the President of the Russian
Federation Vladimir Putin in 2015 to support the economic development of Russia’s Far
East and to expand international cooperation in the Asia-Pacific region.

Hence, option (c) is correct.

https://forumvostok.ru/en/about-the-forum/

www.insightsactivelearn.com 63
© Insights Active Learning | All rights reserved - 42423. You may not reproduce, distribute or exploit the contents in any form without
written permission by copyright owner. Copyright infringers may face civil and criminal liability
Total Marks : 200
Online Prelims TEST - 8 (SUBJECT WISE)
( InsightsIAS Mock Test Series for UPSC Preliminary Exam 2020 ) Mark Scored : 88.67

https://indianexpress.com/article/explained/explained-why-the-eastern-economic-forum-matters-for-i
ndia-5969748/

99 Consider the following statements:


1. Measles is a bacterial disease
2. Rubella is caused by Virus
3. WHO South East Asia Region resolved to eliminate Measles and Rubella by 2030

Which of the statements given above is/are correct?


A. 1 only
B. 2 only
C. 2 and 3 only
D. 1, 2 and 3

Your Answer : C
Correct Answer : B
69
6 55
Answer Justification : 0
9 68
- 8preventable by a vaccine. Hence,
Measles is a childhood infection caused by a virus but is easily
statement 1 is incorrect. om
ail.c
gm
Rubella is a contagious viral infection preventable by vaccine and best known by its distinctive

2@
red rash. Hence, statement 2 is incorrect.
4
0
yHealth
Member-countries of the World
a j a Organisation (WHO) South-East Asia Region have
i
so3nis incorrect.
resolved to eliminate highly infectious childhood killer diseases measles and rubella by

i-
2023. Hence, statement
n
y So
a
Aj to SARAL – ‘State Rooftop Solar Attractiveness Index’, consider the following
100 With reference
statements:
1. It evaluates Indian states based on their attractiveness for rooftop development.
2. Gujarat has been placed at the first rank.

Which of the statements given above is/are correct?


A. 1 only
B. 2 only
C. Both 1 and 2
D. Neither 1 nor 2

Your Answer :
Correct Answer : A

Answer Justification :

SARAL – ‘State Rooftop Solar Attractiveness Index’ was recently launched.

www.insightsactivelearn.com 64
© Insights Active Learning | All rights reserved - 42423. You may not reproduce, distribute or exploit the contents in any form without
written permission by copyright owner. Copyright infringers may face civil and criminal liability
Total Marks : 200
Online Prelims TEST - 8 (SUBJECT WISE)
( InsightsIAS Mock Test Series for UPSC Preliminary Exam 2020 ) Mark Scored : 88.67

The Index evaluates Indian states based on their attractiveness for rooftop development.
Hence, statement 1 is correct.

SARAL is the first of its kind index to provide a comprehensive overview of state-level
measures adopted to facilitate rooftop solar deployment.

SARAL has been designed collaboratively by the Ministry of New and Renewable Energy
(MNRE), Shakti Sustainable Energy Foundation (SSEF), Associated Chambers of
Commerce and Industry of India (ASSOCHAM) and Ernst & Young (EY).

Karnataka has been placed at the first rank. Telangana, Gujarat and Andhra Pradesh have got 2nd,
3rd and 4th rank respectively. Hence, statement 2 is incorrect.

69
6 55
0
9 68
-8
m
i l .co
m a
2 @g
y 04
ia ja
n
- so
n i
o
a yS
Aj

www.insightsactivelearn.com 65
© Insights Active Learning | All rights reserved - 42423. You may not reproduce, distribute or exploit the contents in any form without
written permission by copyright owner. Copyright infringers may face civil and criminal liability

You might also like